You are on page 1of 197

CAPITAL GAINS

CA. VIVEK DOSHI

18 September 2009

IPCC - Income Tax

7.1 CHARGE UNDER THE HEAD CAPITAL GAINS [Section 45]


Capital gains are charged to tax by virtue of Section 45 of the Income Tax Act, 1961. Capital gains mean the profits or gains arising from the transfer of a capital asset. According to Section 45, the charge of income under the head Capital Gains arises if the following conditions are fulfilled: 1. 2. 3. 4. 5. There is a capital asset. [The asset must be a capital asset at the time of transfer] There is a transfer of such capital asset. The transfer of such capital asset has been effected during the previous year. Profits or gains arise from the transfer of such capital asset affected during the previous year. (Profit or gain includes negative profit or gain i.e. loss also) Such profits or gains are not exempt from tax u/s 54, 54B, 54D, 54EC, 54F, 54G and 54H.
IPCC - Income Tax 2

18 September 2009

CAPITAL ASSET [Section 2(14)]:


According to Section 2(14), capital asset means property of any kind held by an assessee, whether or not connected with his business or profession, but does not include Any stock-in-trade, consumable stores or raw materials held for purpose of his business or profession. Personal effects i.e. movable property (including wearing apparel and furniture but excluding jewellery, archaeological collections, Drawings, Paintings, Sculptures and any work of art) held for personal use by assessee or his family member dependent on him. Jewellery is a capital asset. It includes
Ornaments made of gold, silver, platinum or any other precious metal or any alloy containing one or more of such precious metals, whether or not containing any precious or semi-precious stones and whether or not worked or sewn into any wearing apparel; Precious or semi-precious stones whether or not set in any furniture, utensil or other article or worked or sewn into any wearing apparel.
IPCC - Income Tax 3

1. 2.

A.

B.

18 September 2009

CAPITAL ASSET [Section 2(14)]:


3. Rural agricultural land i.e. Agricultural land in India not being a land situated C. Within the jurisdiction of a municipality or a cantonment board having a population of 10,000 or more according to the last preceding census; or D. In any notified area within 8 kms from the local limits of any municipality or cantonment board. Gold Bonds issued by Central Government including the Gold Deposit Bonds issued under the Gold Deposit Scheme, 1999. Special Bearer Bonds, 1991.

4. 5.

18 September 2009

IPCC - Income Tax

1) Capital gains or not : Compute the capital gains in following independent cases for Assessment year 2009-10 1)

2)

3)

4)

5)

Mr. A, a jewellery dealer, sells jewellery on 1-8-2008 for Rs. 15,00,000, which was acquired on 1-8-2007 for Rs. 10,00,000. Mr. B sells his personal furniture on 1-6-2008 for Rs. 50,000, which was acquired on 1-4-2003 for Rs. 80,000. The expenses on transfer are 1% of selling price. Mr. C sells his personal residential house on 1-12-2009 for Rs. 10,00,000, which was acquired by him on 1-4-2003 for Rs. 8,00,000. The expenses on transfer were Rs.22,000. Mr. D sells gold deposit bonds on 1-7-2008 for Rs. 12 lakh. The bonds were acquired on 1-3-1999 for Rs. 8,50,000. Mr. E, a painter, sells the drawings and paintings made by him for Rs. 50 lakhs on 1-10-2008.
IPCC - Income Tax 5

18 September 2009

1) Solution : Capital gains for Assessment year 2009-10 in respect of above cases shall be computed as under Jewellery sold by a jewellery dealer is stock-in-trade of the business. Therefore, no capital gain is chargeable to tax. 2) Personal furniture sold by Mr. B is a personal effect, which is a not a capital asset. 3) The transfer of the house was effected on 1-12-2009, which doesn't fall within the previous year 2008-09. Hence, this transaction is not liable to capital gains in the assessment year 2009-10. It will be liable to capital gains in the assessment year 2010-11. 4) Gold deposit bonds sold by Mr. D are not a capital asset. 5) In case of painter, the selling of drawings and paintings Made by him takes place in the exercise of his profession of painting. Hence, such drawings and paintings are 'stock-intrade' held for the purpose of his profession and are, 6 IPCC - Income Tax 18 September 2009 therefore, not a capital asset.
1)

2)Compute the capital gains in following independent cases for Assessment year 2009-10.
1.

2.

3.

4.

5.

6.

Mr. A, a property dealer, sells a commercial plot of land on 1-8-2008 for Rs 25,00,000, which was acquired on 1-8-2003 for Rs. 10,00,000 for selling of office constructed therein. He had incurred land development charges of Rs. 1,00,000 on 1-4-2006. He incurred Rs 10,000 for selling the plot of land. Mr. B sells his personal motorcar on 1-6-2008 for Rs. 1,00,000, which was acquired on 1-4-2003 for Rs. 3,00,000. The expenses on transfer are 1% of selling price. Mr. C sells his personal residential house on 1-5-2009 for Rs. 20,00,000, which was acquired by him on 1-4-2004 for Rs. 8,00,000. The expenses on transfer were Rs. 15,000. Mr. D sells an agricultural land situated in the Village Rampur on 1-7-2008 for Rs. 20 Lakh. The population of municipality under which the village Rampur is covered is 9,000. The land was acquired on 1-3-1997 for Rs. 50,000. Mr. E sells his gold bracelet on 1-5-2008 for Rs. 4,00,000 which was acquired for Rs. 20,000 on 1-3-1985. A diamond was fitted on to that bracelet on 1-72004 for Rs. 10,000. Mr. F, a businessmen, had acquired a paint in for Rs. 10 lakh and a drawing for Rs. 5 lakh in the year 1988-89. He sold the painting for Rs. 20 lakh to a university and the drawing for Rs. 1 lakh to a National Museum on 5-8-2008.
18 September 2009 IPCC - Income Tax 7

SOLUTION 2 - Capital gains for the assessment year 2009-10 in respect of above cases shall be computed as follows Commercial plot of land acquired by a property dealer for his business purposes is stock-in-trade of the business.Therefore, it was not a capital asset. Personal motorcar sold by Mr. B is a personal effect, which is not a capital asset. The transfer of the house was affected on 1-5-2009, which doesnt fall within the previous year 2008-09. Hence, this transaction is not liable to capital gains in the assessment year 2009-10. It will be liable to capital gains in the assessment year 2009-10.

1.

2.

3.

18 September 2009

IPCC - Income Tax

SHORT TERM AND LONG TERM NATURE OF CAPITAL ASSETS:


(1) Short-term Capital Asset [Section 2(41A)] : Short-term Capital Asset means a capital asset held by assessee for not more than 36 months immediately preceding the date of its transfer. However, in case of (a) Equity or Preference Shares in a company

These assets shall be treated as short-term (b) Other securities listed in recognized stock capital assets if they exchange in India are held for not more (c) Units of UTI or Units of mutual fund specified than 12 months u/s 10(23D) immediately preceding (d) Zero Coupon Bonds the date of transfer.
Note: An asset held exactly for 36 months or 12 months, as the case may be, will also be a short-term capital asst. For computing the period of 36 months or 12 months, as the case may be, the date on which the asset was acquired is to be included while the date on which the asset is transferred is to be excluded.
18 September 2009 IPCC - Income Tax 9

SHORT TERM AND LONG TERM NATURE OF CAPITAL ASSETS:


(2) Long-term Capital Asset [Section 2(29A)]: Any capital asset other than a short-term capital asset is a long-term capital asset. In other words, a capital asset held for more than 36 months (12 months in case of specified assets given in table above) shall be a longterm capital asset.

18 September 2009

IPCC - Income Tax

10

SHORT TERM AND LONG TERM NATURE OF CAPITAL ASSETS:


(3) Determination of Long-Term or Short-Term Nature of a Capital Asset: In determining the short-term or long-term nature of a capital asset, the period of holding shall be determined as follows:
Mode Determination of period of holding

1. Shares held in a company Any period subsequent to the date on which the in liquidation company goes into liquidation shall be excluded 2. Assets acquired Section 49(1) modes under Period for which the asset was held by the previous owner shall be included

3. Share(s) in Indian Period, for which the shares in the amalgamating amalgamated company, company were held by the assessee, shall be which became property of included. assessee in amalgamation
18 September 2009 IPCC - Income Tax 11

4. Bonus securities 5. Right securities

shares

or

other Period of holding will start from the date of allotment thereof. other Period of holding will start from the date of allotment thereof.

shares

or

6. Right renounced

entitlements Period of holding taken from date of offer made by company. Period for which such person was member of Recognised Stock Exchange (RSE) in India prior to such demutualisation or corporatisation shall be included.

7. Equity Shares in a company, or Trading or clearing rights of a RSE; acquired pursuant to demutualisation or corporatisation of such RSE

8. Shares of resulting Period of holding of shares in demerged company company acquired in case of shall be included. 07926401801/02 demerger
18 September 2009 IPCC - Income Tax 12

9. Asset which was not held Entire period of holding from date of initial as capital asset initially acquisition upto date of transfer will be but is a capital asset at considered to decide nature of capital asset. [Keshavji Karsondas v. CIT [1994] 207 ITR the time of transfer. 737 (Bom.]

10.Capital asset being a flat Period of holding to be calculated from date of allotted to a member of a allotment of shares in society and not from co-operative housing date on which possession of the flat is obtained because right of possession and use society of flat is an incidental right flowing from the ownership of shares. [CIT v. Jin-Das PanChand Gandhi [2005] 279 ITR 552 (Guj.)]

18 September 2009

IPCC - Income Tax

13

SHORT TERM AND LONG TERM NATURE OF CAPITAL ASSETS: Property constructed on a land purchased earlier: if land is held by the assessee for more than 36 months and building constructed over it is held for not more than 36 months, in that case, the gains arising from the sale of the land would be long-term capital gains, and gains arising from sale of building will be short term capital gains. The Central board of direct taxes has issued Circular no. 704 dated 28.4.95 clarifying as follows: A. If securities are transacted through stock exchanges, the date of brokers note should be treated as the date of transfer provided the transaction is followed up by delivery of shares and also the transfer deeds. Similarly, in respect of the purchase of the securities, the holding period shall be reckoned from the date of the brokers note for purchase on behalf of the investors. B. In case the transaction take place directly between the parties and not through stock exchanges, the board has clarified that the date of contract of sale as declared by the parties shall be treated as the date of transfer provided it is followed up by actual delivery of shares and the transfer deeds.
18 September 2009 IPCC - Income Tax 14

SHORT TERM AND LONG TERM NATURE OF CAPITAL ASSETS: C. In cases where the shares are purchased in several lots at different points of time and the delivery of which are taken in one lot and subsequently sold in parts, in the absence of correlation of the dates of purchase and sale through specific numbers of the scripts, it is difficult to determine the period of holding of the shares which are sold in parts. In this regard, board has clarified that first-in-first-out (FIFO) method shall be adopted to reckon the period of holding. Therefore, the shares acquired first will always be treated as sold first and the shares acquired last will be taken to be remaining with the assessee. This CBDT has issued an exclusive circular no.768 dated 2406-1998 (232 ITR 5 St.) in respect of transactions in securities held in dematerialized form u/s.45 (2A) for determination of date of transfer and period of holding as detailed below: A. The FIFO method will be applied only in respect of the dematerialized holdings, because in the case of sale of dematerialized securities, the securities held in physical form cannot be construed to have been sold as they continue to remain in the possession of the investor and are identified separately.
18 September 2009 IPCC - Income Tax 15

SHORT TERM AND LONG TERM NATURE OF CAPITAL ASSETS: B. In the depository system, the investor can open and hold multiple accounts. In such a case, where an investor has more than one security account, the FIFO method will be applied account wise. This is because in case where a particular account of an investor is debited for sale of securities, the securities lying in his other account cannot be construed to have been sold as they continue to remain in that account. C. If in an existing account of dematerialized stock, old physical stock is dematerialized and entered at a later date, under the FIFO method, the basis for determining the movement out of the account is the date of entry into the account.

18 September 2009

IPCC - Income Tax

16

SHORT TERM AND LONG TERM NATURE OF CAPITAL ASSETS: Notes :A. Modes specified in Section 49(1): Where the capital asset became the property of the assessee a. On any distribution of assets on the total or partial partition of a Hindu Undivided Family; b. Under a gift or will; or by succession, inheritance or devolution; or c. On any distribution of assets on the liquidation of a company; or d. Under a transfer to a revocable or an irrevocable trust, or Under any such transfer as is referred to in clause (iv)/ (v)/ (vi)/ (via)/ (viaa) of Section 47;

18 September 2009

IPCC - Income Tax

17

TRANSFER [Section 2(47)]:


Transfer, in relation to capital asset, includes a. b. c. d. e. f. sale, exchange or relinquishment of the asset; extinguishment of any rights therein; compulsory acquisition thereof under any law; maturity or redemption of zero coupon bond; conversion or treatment of such asset by the owner into stock in trade of business carried on by him; Any transaction involving allowing of possession of an immovable property to be taken or retained in part performance of a contract of the nature referred u/s 53A of Transfer of Property Act, 1882. any transaction (whether by way of acquiring shares in, or by way of becoming a member of, a co-operative society, company or other AOP or by way of any arrangement or agreement or in any other manner) that has the effect of transferring or enabling the enjoyment of, any immovable property.
IPCC - Income Tax 18

g.

18 September 2009

TRANSFER [Section 2(47)]:


-:Case Laws:1. Reduction in face value of shares and consequent payment to the shareholder towards such reduction amounts to transfer, as it results in extinguishment of right in the shares held by the shareholder. Kartikeya Sarabhai v. CIT [1997] 228 ITR 163 (SC). 2. Surrender of Preference Shares on redemption thereof amounts to transfer as there is relinquishment by the shareholder of his rights in Preference Shares. Anarkali Sarabhai v. CIT [1997] 224 ITR 422 (SC). 3. Family arrangement entered into by compromising doubtful/disputed rights for preserving family property/peace, doesnt amount to transfer. CIT v. A.L. Ramanathan [2000] 245 ITR 494 (Mad.)

18 September 2009

IPCC - Income Tax

19

TRANSACTIONS THAT ARE NOT REGARDED AS TRANSFER [Section 47]:


1. Distribution of assets by a company to its shareholders on its liquidation. [S. 46(1)] 2. Distribution of capital assets on total or partial partition of HUF. [S. 47(i)] 3. Capital asset transferred under will or gift or an irrevocable trust. [S. 47(iii)] However, transfer under a gift or an irrevocable trust of shares, debentures or warrants allotted to the assessee under Employee Stock Option Plan as per prescribed guidelines, shall constitute transfer. Its fair market value on date of such gift/irrevocable trust shall be treated as full value of consideration. 4. Transfer of a capital asset (not held as stock in trade) by a holding company to its 100% subsidiary company or vice versa, provided the transferee company is Indian company. [S. 47(iv)/(v)] 5. Transfer of capital asset by an amalgamating company to Indian amalgamated company. [S. 47(vi)]

18 September 2009

IPCC - Income Tax

20

6. Transfer of share(s) held in an Indian company by amalgamating foreign company to amalgamated foreign company if (a) at least 25% of shareholders of the amalgamating foreign company continue to remain shareholders of the amalgamated foreign company; and (b) such transfer does not attract capital gains tax in the country in which the amalgamating company is incorporated. [S. 47(via)] 7. Transfer of capital asset by an amalgamating banking company to the amalgamated banking institution, under a scheme of amalgamation sanctioned by the Central Government. [S. 47(viaa)] 8. Transfer of capital asset by a demerged company to the resulting company. [S.47(vib)] 9. Transfer of share(s) held in an Indian company by demerged foreign company to the resulting company if (a) shareholders holding 75% or more of value of shares of demerged foreign company continue to remain shareholders of resulting foreign company; and (b) such transfer does not attract capital gains tax in the country in which demerged foreign company is incorporated. [S.47(vic)] 10.Any transfer or issue of shares by resulting company, in a scheme of demerger to the shareholders of the demerged company in consideration of demerger. [S.47(vid)]
18 September 2009 IPCC - Income Tax 21

11.Transfer of share(s) held by shareholder in amalgamating company, if such transfer is in consideration of allotment to him of share(s) in the amalgamated Indian company. (S.47(vii) However, if besides share(s) in amalgamated company, the shareholder is allotted something more, say bonds or debentures, in consideration of such transfer; the transfer will not be exempt. Composite consideration is not covered by Section 47(vii). CIT v. Gautam Sarabhai Trust [1988] 173 ITR 216 (Guj.)] 12.Any transfer, in an amalgamation/demerger, of a capital asset by the predecessor co-operative bank to the successor cooperative bank. [s.47 (vica)]. 13.Any transfer by shareholder, in an amalgamation/ demerger, of share (s) held by him in predecessor co-operative bank if the transfer is made in consideration of the allotment to him of any share (s) in the successor co-operative bank.[S.47(vicb)] 14.Transfer of bonds or Global Depository Receipts [referred to in Section 115AC (1)] of a public sector company made outside India by a non-resident to another non-resident. [S.47(viia)] 15. Conversion of bonds referred to in sec 115 AC (1) (a) into shares or debentures of any company; [S.47 (xa)] (inserted by the Finance act, 2008 w.r.e.f 1-4-2008).
18 September 2009 IPCC - Income Tax 22

16. Transfer of any work of art, archaeological, scientific or art collection, book, manuscript, drawing, painting, photograph or print, to Government/ University/National Museum/National Art Gallery/National Archives or any other notified public institution/museum. [S. 47(ix)] 17. Conversion of bonds or debentures, debenture-stock or deposit certificates in any form, of a company into shares or debentures of that company. [S.47(x)] 18. Transfer of land of sick industrial company (being managed by workers co-operative) made under scheme prepared u/s 18 of Sick Industrial Companies Act, 1985, if such transfer is made during the period starting from previous year in which such company has become sick and ending with the previous year during which its entire net worth becomes equal to or exceeds accumulated losses. [S. 47(xii)]

18 September 2009

IPCC - Income Tax

23

19. Transfer of (a) a capital asset or intangible asset by a predecessor firm to its successor company; or (b) a capital asset to successor company in course of demutualisation/corporatisation of predecessor recognized stock exchange in India (being an Association of Persons or Body of Individuals) [S. 47(xiii)] A. All the assets and liabilities of the firm/AOP/BOI relating to their business immediately before the succession become the assets and liabilities of the company; B. In case of firm, all its partner become shareholders of the company in the same proportion in which their capital accounts stood in the books of the firm on the date of the succession; C. In case of firm, the partners receive consideration only by way of allotment of shares in company. D. In case of firm, the partners shareholding in the company in aggregate is 50% or more of its total voting power and continue to be as such for 5 years from the date of succession; and E. The demutualisation or corporatisation of a recognized stock exchange in India is carried out in accordance with a `scheme for demutualisation or corporatisation, which is approved by the SEBI.

18 September 2009

IPCC - Income Tax

24

20. Transfer of a membership right held by a member of a recognized stock exchange in India for acquisition of shares and trading or clearing rights acquired by such member in that stock exchange in accordance with demutualisation or corporatisation scheme approved by the SEBI. [S.47 (xiiia)] 21. Transfer of capital asset or intangible asset to the successor company by its predecessor proprietary concern, if the following conditions are fulfilled [S.47 (xiv)] A. All the assets and liabilities of the sole proprietary business immediately before the succession become the assets and liabilities of the company. B. Sole proprietors shareholding in the company is 50% or more of the total voting power and continues to be as such for 5 years from the date of succession; and C. The sole proprietor receives the consideration only in form of allotment of shares in the company. 22. Any transfer under Securities Lending Scheme, 1997 for lending of securities under an agreement or arrangement, which is entered into by the assessee with borrower of such securities and which is subject to guidelines issued by SEBI or RBI. [S.47 (xv)]

18 September 2009

IPCC - Income Tax

25

Note: In respect of Section 47(xiii) and 47(xiv), the exemption is available only in respect of the firm/sole proprietor carrying on a business, not in case of profession. Further, this exemption is available only in respect of transfer of capital assets or intangible assets, not in respect of any stock in trade. 23. Any transfer of a capital asset in a transaction of reverse mortgage under a scheme made and notified by the central government[S.47(xvi)](inserted by the Finance act, 2008 w.r.e.f 14-2008).

18 September 2009

IPCC - Income Tax

26

WITHDRAWAL OF EXEMPTION:
Where the capital gain arising on the transfer of a capital asset from the holding company to the subsidiary company or vice-versa was exempt from capital gains tax by virtue of Sec.47 and if any other following events occur within a period of 8 years from the date of transfer, the capital gains so exempted would be chargeable to tax in the year in which the transfer took place i) The holding company does not continue to hold the whole of the share capital of the subsidiary company; ii) The transferee company converts or treats the capital asset into/as stock- in- trade. In the case of a transaction between holding company and subsidiary company, the following additional points need to be borne in mind: a) If the provisions of section 47 are applicable to a transfer, then the assessment shall be reopened in respect of the A.Y. relevant to the previous year in which original transfer took place u/s.155 (7B), to amend the order so as to charge the capital gains to tax. b) if the transferee company subsequently sells the asset without attracting the provision of section 47A, then for computation of capital gains the cost to the transferor company shall be adopted as cost to the transferee company- sec 49 (1).
18 September 2009 IPCC - Income Tax 27

c) if the asset is sold after attracting the provisions of section 47A, then the cost to the transferee company shall be the actual cost incurred by that company to acquire the asset from the transferor company-sec.49(3). The capital gain arising on transfer of a capital asset in the nature of membership of a recognized stock exchange exempted by virtue of sec.47, shall be chargeable to tax if the shares allotted to the transferor in exchange thereof are transferred before the expiry of a period of 3 years. The capital gain shall be deemed, in such a case, as the income chargeable during the previous year in which the shares are transferred. If the conditions stipulated regarding the succession of a proprietary concern or a firm by a company are not complied with, the benefits availed by the sole proprietor or the firm, as the case may be, shall be deemed to be profit and gains of the successor company chargeable to tax in the year in which infringement takes place.

18 September 2009

IPCC - Income Tax

28

COMPUTATION OF CAPITAL GAINS SHORT TERM AND LONG TERM


Short term capital gains [S. 2(42B)] means capital gains arising from transfer of a short-term capital asset. Long term capital gains [S. 2(29B)] means capital gains arising from transfer of a long-term capital asset. Mode of Computation of Capital Gains [Section 48]
Short Term Capital Gains Full Value of Consideration Less : Exp incurred wholly and exclusively for such transfer Net Consideration Less : *C.O.A. **C.O.I. XX XX XX XX XX XX XX XX XX Long Term Capital Gains Full Value of Consideration XX Less : Expenses incurred wholly and XX exclusively for such transfer Net Consideration Less : Indexed * C.O.A. Indexed**C.O.I. XX XX XX XX

Short term capital gain Less : Exemption u/s 54B, 54D, 54G, 54GA Taxable Short Term Capital Gain
18 September 2009

Long term capital gain XX Less : Exemptions u/s 54, 54B, 54D, 54EC, 54F, 54G, 54GA XX Taxable Long Term Capital Gain XX

*Cost of Acquisition (C.O.A) ** Cost of Improvement (C.O.I)


IPCC - Income Tax 29

Notes:
Any sum paid on account of securities transaction tax is not deductible in computing Capital Gains. 2. Indexed cost of acquisition or improvement shall be computed as follows : Cost of acquisition or improvement Cost Inflation index of the year of transfer Indexed Cost of Acquisition = Cost Inflation Index (CII) for (i) the first year in or which the asset was held by the assessee or for the Improvement year beginning on 1.4.1981, whichever is later, or (ii) the year in which improvement took place 1.

18 September 2009

IPCC - Income Tax

30

-:Cost Inflation Indices:The cost inflation indices as notified by the Central Government are as follows:

F. Y. 1981-82 1982-83 1983-84 1984-85 1985-86 1986-87 1987-88

CII 100 109 116 125 133 140 150

F.Y. 1988-89 1989-90 1990-91 1991-92 1992-93 1993-94 1994-95

CII 161 172 182 199 223 244 259

F.Y. 1995-96 1996-97 1997-98 1998-99 1999-2000 2000-01 2001-02

CII 281 305 331 351 389 406 426

F. Y. 2002-03 2003-04 2004-05 2005-06 2006-07 2007-08 2008-09

CII 447 463 480 497 519 551 582

18 September 2009

IPCC - Income Tax

31

C.O.A AND C.O.I. IN CERTAIN CASES [Section 49 and 55]


1. Cost of Acquisition (COA) and Cost of Improvement (COI) in case of a capital asset acquired before 1.4.1981:
Mode of Acquisition COA COI

Where the assessee himself FMV on 1.4.1981 or cost Capital expenditure acquired the capital asset before of property, whichever is incurred by the previous 1.4.1981 higher owner or the assessee in making any additions/ alterations to the capital Capital asset acquired by Cost to the previous asset on or after or FMV on assessee under any of the modes owner 1.4.1981. given in Section 49(1) and the 1.4.1981 whichever is previous owner acquired the higher. same before 1.4.1981

18 September 2009

IPCC - Income Tax

32

C.O.A AND C.O.I. IN CERTAIN CASES [Section 49 and 55]


2. Cost of Acquisition and Improvement in some special cases:
Mode 1. Shares held in a company in liquidation. Cost of Acquisition or Improvement Actual cost of acquisition of such shares.

2. Assets acquired under any of the modes Cost = Cost to previous owner + Cost of specified in Section 49(1) improvement incurred by previous owner or assessee Share(s) in Indian amalgamated company, which becomes the property of assessee in a scheme of amalgamation. Cost of acquisition of shares in amalgamated company = Cost of acquisition of the shares in the amalgamating company [Sec. 49(2)]

3.

18 September 2009

IPCC - Income Tax

33

C.O.A AND C.O.I. IN CERTAIN CASES [Section 49 and 55]


Mode 4. Conversion of bonds or debentures, debenture-stock or deposit certificates in any form, of a company into shares or debentures of that company. Cost of Acquisition or Improvement Cost of acquisition of new shares or debentures = Total cost of bonds, debenture, debenture-stock or deposit certificates Part of such bonds, debenture, debenture-stock or deposit certificates so converted [Sec. 49(2A)]

5.Conversion of bonds or debentures, debenture-stock on deposit certificates in any form, of a company into shares or debentures of that company (i.e. exempt transfers referred u/s 47(x) & 47(xa))

Cost of acquisition of new shares or debentures = total cost of bonds, debentures, debenture-stock or deposit certificates * part of such bonds, debenture, debenture-stock or deposit Certificates so converted [sec.49A(2A)] [Amdt. by Finance Act, 08 w.r.e.f 1-4-08]

18 September 2009

IPCC - Income Tax

34

C.O.A AND C.O.I. IN CERTAIN CASES [Section 49 and 55]


Mode 6. Bonus shares or other securities Cost of Acquisition or Improvement If allotted before 1.4.1981, Cost = Fair market value as on 1.4.1981, otherwise cost = Nil. If purchased by original shareholder : Cost = Purchase Price If purchased by person in whose favour right was renounced : Cost = Purchase Price paid to company + Amount paid for renouncement in his favour Cost = NIL Cost of shares in resulting company = Cost of shares in demerged company Net Book Value of assets transferred to resulting company Net worth of the company before demerger. Cost of shares in demerged company = Total cost of shares Cost of shares in resulting company computed above. 35 IPCC - Income Tax

7. Right shares or other securities

8. Rights entitlements renounced 9. Shares of resulting company acquired in case of demerger

18 September 2009

C.O.A AND C.O.I. IN CERTAIN CASES [Section 49 and 55]


10. Equity Shares & trading/clearing rights Cost of Equity Shares = Cost of acquisition of membership card of stock exchange. in recognized stock exchange acquired on demutualisation/ corporatisation Cost of trading or clearing rights = NIL thereof 11. Share/stock of company acquired on Cost of acquisition of such share or stock = Cost calculated with reference to the (a) Consolidation & division of share capital into shares of larger or smaller cost of acquisition of the shares or amount, (b) conversion of shares into stock from which such share or stock is stock or vice versa, (c) conversion of derived. one kind of shares in other 12.Shares Acquired under an ESOP Cost of acquisition of such share or stock = scheme or acquire as sweat equity Fair Market Value which has been shares taken into account while computing value of Fringe Benefits u/s 115WC(i)(ba)

18 September 2009

IPCC - Income Tax

36

C.O.A AND C.O.I. IN CERTAIN CASES [Section 49 and 55]


3. Cost of acquisition and cost of improvement in case of certain intangible assets
Capital asset being Goodwill of business, right to manufacture/produce/process any article/thing, or right to carry business COI NIL COA If self-generated: Nil. If purchased from previous owner : Purchase Price

Trademark/brand name associated with business or tenancy rights or stage carried permits/loom hours

Expenses incurred by assessee or previous owner after 31.3.1981

18 September 2009

IPCC - Income Tax

37

C.O.A AND C.O.I. IN CERTAIN CASES [Section 49 and 55]


4. Cost of Improvement in any other case: Cost of improvement means
all capital expenditure incurred in making any additions or alterations to capital asset by the assessee after it became his property, and where capital asset became property of the assesee by any of modes specified u/s 49(1), by the previous owner. Exclusions from Cost of Improvement: Cost of improvement does not include any expenditure, which is deductible in computing the income chargeable under the head Income from House Property, Profits and Gains of Business or Profession, or Income from Other Sources. Notes: (A) In case of HUF-assessee, by conversion of members individual property into HUF property. (B) Previous Owner: Previous Owner means the last previous owner of the asset who acquired it by a mode of acquisition other than that referred to under Section 49(1). (C) When cost to previous owner not ascertainable [Sec. 55(3)] : Where the cost for which the previous owner acquired the property cannot be ascertained, the cost of acquisition to the previous owner means the fair market value on the date on which the capital asset became the property of the previous owner.
IPCC - Income Tax 38

18 September 2009

C.O.A AND C.O.I. IN CERTAIN CASES [Section 49 and 55]


5.

Indexed cost of acquisition v/s. indexed cost of improvement:


It needs mention that in the case of assets acquired in any of the modes specified in section 49 (1), the benefit of indexation for cost of acquisition can be claimed only from the first year in which the asset was held by the assessee. However, in the case of indexation of cost of improvement, the benefit of indexation can be availed from the year in which improvement to the asset was made.

6. Conversion of debentures into shares:


Similarly, if debentures are converted into shares, it is not regarded as transfer by virtue of section 47(x). If these shares are sold subsequently, the cost of acquisition would be the cost incurred to acquire the debentures on conversion of which the shares were obtained as provided in section 49 (2A). Nevertheless, there is no provision to enable the assessee to take the period of holding of the debentures in determination of the long-term nature of the shares and again the possibility of claiming the indexation benefit for the period for which debentures were held is ruled out.

18 September 2009

IPCC - Income Tax

39

C.O.A AND C.O.I. IN CERTAIN CASES [Section 49 and 55]


7. Conversion of investment into stock -in- trade:
in the case of conversion of capital asset in to stock-in-trade the provisions of sec.45 (2) explicitly provide for deferring the chargeability till the year of sale of stock-in-trade. While computing the capital gains of sale of the stock-in-trade, the assessee will have to index the cost of acquisition only up to the year of conversion and not up to the year of chargeability since indexation stops in the year of transfer and does not extend to the year in which the computation is made and taxability arises.

8. Compulsory acquisition:
Again, when compulsory acquisition is the instance of transfer in the assessee's case, section 45(5) provides for charging the capital gain only in the year of receipt of the compensation and not in the year of compulsory acquisition. Nevertheless, indexation benefit would not run up to the year of receipt of compensation but would be confirmed only up to the year of compulsory acquisition.

18 September 2009

IPCC - Income Tax

40

C.O.A AND C.O.I. IN CERTAIN CASES [Section 49 and 55]


Case Laws: 1. Amount paid to clear mortgage: Where property has been
mortgaged by previous owner during his life-time and the assessee, after inheriting the same, has discharged mortgage debt, then by discharging the mortgage debt, the assessee acquires the interest of the mortgagee in the property. The amount so paid shall be treated as cost of acquisition. R.M. Arunachalan v. CIT [1997] 227 ITR 222 (SC). However, where after acquiring a property, the assessee himself created a mortgage and cleared off the same out of sale proceeds of property, he couldnt be allowed deduction of payment of mortgage debt as cost of acquisition/ improvement u/s 48 because in that case, he did not acquire any interest in property subsequent to his acquiring the same. VSMR Jagdishchandran v. CIT [1997] 227 ITR 240 (SC)

18 September 2009

IPCC - Income Tax

41

C.O.A AND C.O.I. IN CERTAIN CASES [Section 49 and 55]


2. Kist deducted property: from proceeds of mortgaged
The Government auctioned the mortgaged property of assessee for kist amount due by him to the State, and paid the balance amount (after deducting kist) to the assessee. The assessee claimed deduction for kist amount in computing capital gains. Held that, since the price received in auction entirely belonged to the assessee, the amount deducted towards kist was not diverted at source but was applied in discharge of an obligation after it was received by the assessee. Therefore, kist amount was not deductible in computing capital gains. CIT v. Attili N. Rao [2001] 252 ITR 880 (SC).

18 September 2009

IPCC - Income Tax

42

C.O.A AND C.O.I. IN CERTAIN CASES [Section 49 and 55]


3. No charge, when computation not possible : If, on the facts of a particular case, computation u/s 48 is not possible, then capital gains shall not be charged to tax. Thus, if no cost can be envisaged in acquisition of an asset, capital gains cannot be charged. CIT v. B.C. Srinivasa Setty [1981] 128 ITR 294 (SC). 4. Amount embezzled while effecting sale of property will not constitute expenditure in connection with transfer and is, therefore, is not deductible u/s 48(1) Mr. G.Y. Chenoy v. CIT [1999] 234 ITR 89 (AP). 5. In CIT v/s C.V. Sounderajan, 150 ITR 80(mad) the amount paid to the mother having right of residence in the property, for obtaining relinquishment of such right was held deductible in computing the capital gains.

18 September 2009

IPCC - Income Tax

43

C.O.A AND C.O.I. IN CERTAIN CASES [Section 49 and 55]


6. When Loan is borrowed and invested in any asset, interest expenditure incurred thereon can be claimed as deduction from the income derived from such asset. If the assessee desires to capitalize the interest, is it possible to treat it as part of the cost of acquisition and claim it as deduction in the computation of capital gains is an issue which has been favourably considered by courts. So long as the loan has been exclusively borrowed and utilised for acquisition of an asset, capitalisation of interest is possible as held in the case of CIT v/s Mithlesh Kumari, 92 ITR 9 (DEL) of Addl. CIT v/s K.S.Gupta, 119 ITR 372 (AP). Similar analogy can be inferred from the decisions rendered in CIT v/s Maithreyi Pai, 152 ITR 247 (Kar) and Saharanpur Electric Supply Co. v/s CIT, 194 ITR 294(SC).

18 September 2009

IPCC - Income Tax

44

CASES WHERE BENEFIT OF INDEXATION IS NOT AVAILABLE EVEN IN CASE OF LONG-TERM CAPITAL ASSETS:
1. Transfer of a bond or a debenture other than capital indexed bonds issued by the Government. 2. Transfer of undertaking or division in a slump sale under Section 50B. 3. Transfer of shares/debentures of an Indian company purchased by a non - resident in foreign currency. 4. Transfer of units purchased in foreign currency by an assessee covered under Section 115AB. 5. Transfer of bonds or shares purchased in foreign currency by an assessee covered u/s 115AC. 6. Transfer of global depository receipts by a resident employee of an Indian company u/s 115ACA. 7. Transfer of securities by foreign institutional investors under Section 115AD. 8. Transfer of a foreign exchange asset by a non-resident Indian under Section 115D.
18 September 2009 IPCC - Income Tax 45

3)Compute his capital gain.


Shri RituRaj purchases a house property in Jaipur for Rs. 12,00,000 on 15-10-2007. He constructed first floor on 15-12-2007 and spent Rs. 1,80,000 on such construction. On 15-12-2008 he sold this house for Rs 22,00,000. The expenses on transfer were Rs. 29,000.

18 September 2009

IPCC - Income Tax

46

3) Solution :
Computation of capital gains of Shri RituRaj (Amount in Rs.)

Full value of consideration Less: Expenses on transfer Less: Cost of acquisition Less: Cost of improvement incurred after 1-41981 Short term Capital Gains 29,000 1,200,000 180,000

2,200,000

1,409,000 791,000

Note: Since the period of holding of house property does not exceed thirty-six months hence the capital asset is a short-term capital asset.

4)Compute his capital gain.


Shri Ramesh purchased a house property in Ganganagar for Rs 6, 50,000 on 28-12-1978. He paid registration fees of Rs 65,000. The fair market value of the property as on 1st April, 1981 was Rs.7,25,000. He constructed first Floor on 10-12-1999 and spend Rs. 3,89,000 on the said construction. On 15-12-2008 he sold this house for Rs 72, 00,000. The expenses on transfer were Rs. 72,000.

18 September 2009

IPCC - Income Tax

48

4) Solution :
Computation of Taxable Capital Gains of Shri Ramesh for A.Y. 2009-10 :

Rs. 7,200,000 72,000 4,219,500 582,000 2,326,500


49

Full value of consideration Less: Expenses in connection with transfer Less: Indexed cost of acquisition (725,000 x 582 /100) Less: Indexed cost of improvement (3,89,000 x 582 389) Long term Capital Gains
18 September 2009 IPCC - Income Tax

5)Compute his capital gain.


Shri Prabhat purchased a land in Jaipur for Rs 18,50,000 on 28-12-1998. He paid registration fees of Rs. 1,80,000. On 15-32009 he sold this house for Rs 64,50,000. The expenses on transfer were Rs 64,500.

18 September 2009

IPCC - Income Tax

50

5) Solution :
Computation of Taxable Capital Gains of Shri Prabhat for A.Y. 2009-10 (amounts in Rs.) :
Full value of consideration Less: Expenses in connection with transfer 6,450,000 64,500

Less: Indexed cost of acquisition (20,30,000 x 582 351)

3,365,983

Long term Capital Gains 3,019,517


18 September 2009 IPCC - Income Tax 51

6) Question
Shri Megani purchased a commercial building in Udiapur for Rs 3,31,000 on 27-11-1994. He gifted the property to his friend Chandu on 1812-1997. On 15-1-2009 Chandu sold this house for Rs. 7,80,000. The expenses on transfer were Rs 48,000. Compute capital gains in hands of Shri Chandu and state in which year it shall be chargeable to tax.

18 September 2009

IPCC - Income Tax

52

6) Solution
Computation of capital gain of Shri Chandu for the Assessment Year 2009-10 (amounts in Rs.)

Full value of consideration


Less: Expenses in connection with transfer Less: Indexed cost of acquisition (3,31,000 x 582 331)

780,000 48,000 582,000 150,000

Long term Capital Gains

18 September 2009

IPCC - Income Tax

53

7) Question
Mr. F sells a plot of land on 1-8-2008 for Rs 25,00,000. He inherited the plot from his father on 1-42004. His father had acquired the plot on 1-3-1981 for RS 30,000. His father had incurred land development charges Rs. 10,000 on 31-3-1981 and Rs. 20,000 on 15-2001. Mr. F had incurred land development charges Rs 50,000 on 1-9-2005. The sale stamp deed expenses were 1% of the selling price. The FMV of the plot as on 1-4-1981 was Rs 29,000. Compute the capital gains for the assessment year 2009-10.

18 September 2009

IPCC - Income Tax

54

7) Solution : In case of property acquired in modes specified under section 49(1)

(here, inheritance), the cost to the previous owner or FMV as on 1-4-1981 will be taken. Cost of improvement incurred after 1-4-1981 by previous owner or the assessee, is deductible. For determining long-term or short-term nature of land, the period of holding of Mr. F's father will be included, as a result of which, the plot of land becomes long-term capital asset for Mr. F. For indexation purposes, CII of the year in which property was held by the assessee i.e. year of inheritance being 2004-05 will be taken as denominator.

Full value of consideration Less: Expenses on transfer being sale stamp deed expenses @ 1% of 25,00,000

2,500,000 25,000

Net consideration 2,475,000 Less: Indexed cost of acquisition (30,000 x 582 = 480)
Less: Cost of improvement incurred after 1-4-1981

36,375 87,949

Mr. F's father (20,000 x 582 426) + Mr. F (50,000 x 582 = 480)
18 September 2009 IPCC - Income Tax Long-term

55 capital gains 2,350,676

8. Mr. I, a manufacturer of bricks since 1999, sells the following assets on 12-3-2009, Name of asset Goodwill Trademark Tenancy rights Manufacturing in respect of licence business premises
Self-generated N.A. Rs. 20,000 31-3-2008 5,00,000 Purchased Rs. 30,000 12-3-2006 NIL

Mode of acquiring Purchase price & date thereof Cost of improvement and date thereof price

Self-generated N.A. Rs. 2,00,000 1-4-2007 15,00,000

Purchased Rs. 1,00,000 11-3-2006 Rs. 10,000 1-5-2007 4,00,000

2,00,000

Compute the capital gains for assessment year 2009-10.


18 September 2009 IPCC - Income Tax 56

8) Solution :
The assets transferred in this case are intangible assets, the capital gains for the assessment year 2009-10, on transfer of which are as follows Name of asset Full value of consideration Less: Cost of Acquisition Less: Cost of Improvement Long-term capital gains Short-term capital gains Goodwill 1,500,000 0 0 1,500,000 Trademark 400,000 121,250 11,214 267,536 Tenancy rights 500,000 0 22,428 477,572 170,000 Mfg. licence 200,000 30,000 0

Notes: (1) Trademark is a long-term capital asset as it was held for more than 36 months before date of transfer. However, manufacturing licence was held for exactly 36 months (i.e. not more than 36 months) before date of transfer, hence, it is short-term capital asset. Goodwill and tenancy rights are longterm capital assets as they were held since 1999. (2) In case of goodwill of a business, cost of improvement is always NIL.
18 September 2009 IPCC - Income Tax 57

9) Question
A is a shareholder of X & Co. Ltd., holding 1,000 shares of the face value of Rs. 10 each, allotted at the time of the company's incorporation in May, 1998. The company made a right issue in the ratio of 1:1 on 157-2008 at a premium of Rs. 40 per share. Instead of taking up the right, he renounced it in favour of 'B' at a price of Rs. 10 per share. What is the capital gain chargeable in the hands of 'A'? What will be the cost of the shares in the hands of B? (Nov. 1995, May 2000)
18 September 2009 IPCC - Income Tax 58

9) Solution :
(1) Short-term capital gains in hands of A = Sale proceeds of the right entitlements Cost thereof = 10 x 1000 - Nil = Rs. 10,000. (2) Cost of acquisition for B = Amount paid to A + Amount paid to X Ltd. =10 x 1000 + 50 x 1000 = Rs. 60,000.

18 September 2009

IPCC - Income Tax

59

10) Question
A purchased 250 equity shares of ABC Ltd on 1.4.1993 for Rs. 270 per share and incurs expenditure of Rs. 500 on brokerage and share transfer fees. On 1.7.1997 he gets 200 bonus shares. On 1.9.2003 he gets 300 right shares for Rs. 140 per shares. On 28.2.2009 he sells all the 750 shares for Rs. 400 per share and incurs an expenditure of Rs. 1,500 on brokerage. Compute his taxable income for Assessment Year 2009-10. He does not have any other IPCC - Income Tax 18 September 2009 of income. source (Nov. 1992)60

10) Solution :
Computation of Long Term Capital Gain of A for the Assessment Year 2009-10
Original Shares (250) Sale Consideration Less: Brokerage (1500 Net Consideration
18 September 2009

Right Shares (300) 120,000 600 119,400

Bonus Shares (200) 80,000 400

100,000 750 = 2 per share) 500 99,500


IPCC - Income Tax

79,600

61

11) Question
Mr. A had acquired painting worth Rs. 10 lakhs on 1-6-2005. On same date, he inherited a sculpture and a rare archaeological collection from his father. His father had acquired the sculpture on 1-7-1996 for Rs. 5 lakhs. His father had found the rare archeological collection from the earth underneath his house. He sells all the three things for Rs. 20 lakhs each on 1-6-2008. Compute the amount of capital gains chargeable to tax.
18 September 2009 IPCC - Income Tax 62

11) Solution :
Computation of Capital Gains of Mr. A for the Assessment Year 2009- 10(amounts in Rs.)
Painting Sculpture

Full value of consideration


Less: Cost of acquisition (5,00,000 x 582 + 497)

2,000,000 1,000,000

2,000,000 585,513

Short-term capital gains

1,000,000

Long-term capital gains

1,414,487

Note : As the cost of rare archaeological collection is not ascertainable, there can't be any charge of capital gains on it.
18 September 2009 IPCC - Income Tax 63

SCOPE AND YEAR OF CHARGEABILITY OF CAPITAL GAINS [Section 45]:


S.45 (1) Transaction Transfer of capital asset Full Value of Consideration Year of Chargeability Agreed consideration Previous year in which transfer took place. (subject to Sec.50C and Sec.55A)

(1A)

Damage to, destruction of, capital asset. [Note 1]

or Insurance Previous year in which any compensation i.e. money or other asset is received Money + Fair market from the insurance value (on date of company. receipt) of other assets received

(2)

Conversion of a capital The fair market value as Previous year in which stock in trade is the date of asset into stock in on sold. conversion. trade [Note 2] Transfer of shares held Agreed consideration in depository (FIFO basis)
IPCC - Income Tax

(2A)

Previous year in which transfer took place


64

18 September 2009

SCOPE AND YEAR OF CHARGEABILITY OF CAPITAL GAINS [Section 45]:


(3) Transfer of capital asset as capital contribution or otherwise by a partner or member to Firm/AOP/ BOI year in Amount at which Previous such asset is which transfer took recorded in books of place. the Firm/AOP/BOI.

(4)

Distribution of capital Fair market value as Previous year in asset on dissolution on the date of which transfer took or otherwise of transfer [Note 3] place. Firm/AOP/ Body of Individuals Compulsory acquisition under any law; or any transfer, whose consideration is deemed or approved by Central Govt. or RBI. Compensation awarded; or amount of compensa-tion as determined or approved by Central Government/RBI
IPCC - Income Tax

(5)

The year in which such compensation or part thereof is first received. [Note 4]

18 September 2009

65

Notes: 1. Section 45(1A) applies only when the damage/destruction is due to


a. Flood, typhoon, hurricane, cyclone, earthquake or other convulsion of nature; or b. Riot or civil disturbance; or c. Accidental fire or explosion; or d. Action by enemy or action taken in combating an enemy (whether with or without declaration of war).

However, where damage/destruction is not attributable to any of the reasons aforesaid, there will be no charge of capital gains, as there can be no transfer without existence of capital asset at the time of transfer. Vania Silk Mills P. Ltd. v. CIT [1991] 191 ITR 647 (SC). Computation of capital gains in respect of such assets: As per the CBDTs circular issued in this behalf, capital gains would be worked out in respect of assets which get destroyed, etc. as per the provisions of Sections 48 and 50, as the case may be, by taking the insurance money or the market value of the asset received from the insurer as the full value of consideration. Further, adjustment for cost inflation index will be made for non-depreciable assets and for depreciable assets, the written down value of such assets will be reduced from the block of assets as provided for in Section 43(6).
18 September 2009 IPCC - Income Tax 66

2. In this case, transfer takes place in the year of conversion. So, CII of the year of conversion is used for computation of capital gains. Further, such fair market value will be taken as cost of converted stock. 3. When the partners or members transfer the capital assets, the agreed consideration will be taken as their cost of acquisition. 4.(a)In case of enhanced compensation : In case the compensation is enhanced or further enhanced by the Court, Tribunal or other authority, the capital gains shall be chargeable to tax in the year when the enhanced compensation is received. The amount of enhanced compensation will be the full value of consideration and the cost of acquisition and cost of improvement in that case shall be nil. If the enhanced compensation is received by any other person due to the death of the transferor or due to any other reason, the amount will be deemed to be capital gain of the recipient. (b)Reduction in compensation : In case the initial compensation or the enhanced or further enhanced compensation is reduced by the court or Tribunal or any other authority, such assessed capital gain for that year shall be recomputed by taking the compensation or consideration as so reduced by the court, tribunal or other authority to be the full value of consideration.
18 September 2009 IPCC - Income Tax 67

: Some Issues :
1. Payment, by way of cash or otherwise, to retiring partner over and above balance in his capital account : So far as retiring partner is concerned, the amount received by such partner from the firm in excess of capital and profits standing to his credit cannot be considered as capital gains, as there is no transfer. The amount received by him is not consideration for transfer of his interest to the continuing partners; he only receives his share in partnership. CIT v. R. Lingmallu Raghukumar [2001] 247 ITR 801 (SC). However, so far as the firm is concerned, it has been held in CIT v. A.N. Naik Associates [2004] 265 ITR 346 (Bom), that distribution of asset by the firm to a partner on his retirement shall come within the expression otherwise (as appearing in Section 45(4) and amounts to transfer of capital assets within the meaning of Section 45(4) and therefore, is liable to capital gains tax in the hands of the firm.

18 September 2009

IPCC - Income Tax

68

2. Distribution to partner on dissolution v. Gift of land to Partner: So far as registration is concerned, gift of land to partner is required to be registered under Registration Act, 1908, but the distribution of land to partner on dissolution of the firm, doesnt require registration, as decided in N. Khadervali Saheb v. N. Gudu Sahib [2003] 261 ITR 1 (SC). So far as taxability is concerned, gift of capital asset being a land, is exempt u/s 47(iii), but distribution of land on dissolution is taxable u/s 45(4). Thus, decision as to gift or distribution on dissolution is to be taken after taking this into consideration. 3. Interest on enhanced compensation: Interest received on enhanced compensation in case of compulsory acquisition or the transfer referred to in Section 45(5), will be taxable as income from other sources as per the method of accounting followed by assessee. If assessee follows cash system, it will be taxable in the year of receipt. However, if assessee follows mercantile system, such interest shall be spread on an annual basis over the period right from the date on which asset was acquired to the date on which the order for enhancement is made by the Court. [Rama Bai v. CIT [1990] 181 ITR 480 (SC)].
18 September 2009 IPCC - Income Tax 69

12) Question
Mr. A is an individual carrying on business. His stock and machinery were damaged and destroyed in a fire accident. The value of stock lost (total damaged) was Rs. 6,50,000. Certain portion of the machinery could be salvaged. The opening WDV of the block as on 1-4-2008 was Rs. 10,80,000. During the process of safeguarding machinery and in the fire fighting operations, Mr. A lost his gold chain and a diamond ring, which he had purchased in April, 2005 for Rs. 1,20,000. The market value of these two items as on the date of fire accident was Rs. 1,80,000. Mr. A received the following amounts from the insurance company : 1. Towards loss of stock Rs.4,80,000 2. Towards damage of machinery Rs.6,00,000 3. Towards gold chain and diamond ring Rs.1,80,000 You are requested to briefly comment on the tax treatment of the above three items under the provisions of the Income-tax Act, 1961. (Nov. 2006) (7 Marks)
18 September 2009 IPCC - Income Tax 70

12) Solution :

Assuming that both the breaking out of the fire and receipt of insurance compensation took place place during the previous year 2008-09 and whole of the insurance compensation has been received in cash, the taxability of the aforesaid items has been discussed hereinbelow (i) Treatment of compensation in respect of stock-in-trade : Stock-in-trade is not a capital asset. Loss of stock-in-trade is a trading loss. The same is allowable as deduction in computing income under the head Profits and gains of business or profession. Net loss of stock-in-trade = Value of stock - Insurance money = 6,50,000 - 4,80,000 = Rs. 1,70,000. Alternatively, the whole of the value of stock can be claimed as deduction; and the insurance compensation received can be assesseed as taxable business receipt. (ii) & (iii) Treatment of compensation in respect of machinery, chain and ring : While the machinery has been directly destroyed by fire; the gold chain and diamond ring has been destroyed during fire fighting operation (the immediate cause thereof being the fire). Therefore, the compensation has been received under circumstances referred to in section 45(1A). Accordingly, the capital gains shall be computed as follows IPCC - Income Tax 71

18 September 2009

Machinery

Gold Chain

&
Diamond Ring Full value of consideration (being insurance compensation received) Less : Cost of acquisition being WDV of the block of machinery Indexed cost of diamond & ring (1,20,000 x 582 497) Short-term Capital Loss Long-term Capital Gains
IPCC - Income Tax

600,000 1080,000

180,000

140,523 480000 39,477


72

18 September 2009

13) Question
Pritish, who owns 9 acres of land near Chennai since July 1985 purchased for Rs. 12 lakh, commenced real estate business from April 1999 and introduced this land as his capital. Fair market value of the land on the date of commencement of the business was Rs. 65 lakh. However, the value of such land has been recorded at Rs. 80 lakh in the books of business. The entire land after development and conversion into housing plots was sold for Rs. 135 lakh between September 2008 and February 2009. Expenses incurred for project development were Rs. 37 lakh. Advise him as to taxability of income and under what heads and in which 73 IPCC - Income Tax 18 September 2009 assessment years. (CS Dec. 2005)

The land owned by Pritish is converted into stock-in-trade in the financial year 1999-2000, hence, the year of transfer is 1999-2000. The fair market value on the date of conversion i.e. Rs. 65 lakhs shall be the full value of consideration. However, the capital gains so computed will be taxable in the year of sale of the asset i.e. in the financial year 2008-09 or assessment year 2009-10, as follows -

13) Solution

Computation of taxable income of Pritish for the Assessment year 2009-10 (amounts in Rs.) Full value of consideration (Fair market value of land) Less: Indexed cost of acquisition (12,00,000 x 389 /133) 6,500,000 3,509,774 2,990,226 13,500,000 6,500,000 3,700,000
IPCC - Income Tax Business

Long-term capital gains

Turnover on sale of Plots Less : Cost of Land (FMV) Expenses for project development
18 September 2009

10,200,000
74 3,300,000

income

14) Question
Aarnav converts his plot of land purchased in July, 2001 for Rs. 80,000 into stock - in - trade on 31st March, 2008. The fair market value as on 31.3.2008 was Rs. 1,90,000. The stock-intrade was sold Rs. 2,25,000 in the month of January 2009. Find out the taxable income if any, and if so under which 'head of income' and for which Assessment Year ? CII : F.Y. 20012002 426, F.Y. 2007-2008 551, F.Y. 2008-2009 582. (May 2008) (5 75 IPCC - Income Tax 18 September 2009 Marks)

14) Solution :
Conversion of capital asset into stock-in-trade is transfer exigible to capital gains tax, but the capital gains are charged to tax in the year of sale of stock-in-trade. The tax implications are (in Rs.) Full value of consideration (being FMV on the date of transfer) Less: Indexed cost of acquisition (80,000 x 551/ 426)
Long-term capital gains taxable in assessment year 2009-10

190,000 103,474 86,526

Sale price of stock Less: Cost of stock being FMV as on the date of conversion Business income taxable in assessment year 2009-10

225,000 190,000 35,000

18 September 2009

IPCC - Income Tax

76

Q 15) Mr. Rishi and Manish formed a partnership firm. Just after formation of the partnership, Mr. Rishi brought the following assets into the firm on 15th July 2008 as his capital contribution
House property (Rs.) Ornaments of gold (Rs.)

Market value of the property on the date of transfer Amount recorded in the books of firm Actual cost Year of acquisition

10,80,000 9,00,000 60,000 1984-85

25,000 36,000 15,000 2007-08

What tax consequences will Mr. Rishi have to face in respect of the above transaction? What will be your answer if Mr. Rishi brings the above assets otherwise than by way of his capital contribution?
18 September 2009 IPCC - Income Tax 77

15) Solution Transfer of capital asset by a partner to his firm, by way of his capital contribution or otherwise, is exigible to capital gains tax u/s 45(3) in the hands of the partner in the year in which transfer takes place. Computation of capital gains of Mr. Rishi House property Gold ornament s

Full value of consideration (amount recorded in books of the firm) Less: Cost of acquisition (Indexed cost of house = 60,000 x 582 /125)

900,000
279,360

36,000
15000

620,640

Long-term capital gains


21,000

Short-term capital gains Note: If Mr. Rishi brings the above assets otherwise than by way of capital contribution, then also, he will be liable to capital gains tax as above.
18 September 2009 IPCC - Income Tax

78

Q16) Shri Suresh and X Ltd. are members of SM Associates, an association of persons. SM Association was dissolved on 16th August 2008 and the following assets were distributed to the members (all House property Listed Govt. amount in Rs.)

(given to Suresh) FMV as on 15th August 2008 Amount recorded in agreement of sale Cost of acquisition Date of acquisition FMV of the asset as on 1-4-1981 8,00,000 6,00,000 40,000 1-4-1977 1,00,000

Securities (given to X Ltd.) 6,00,000 7,00,000 5,00,000 16-8-06 -

What are the tax implications of these transactions? What will be your answer if SM Associates is a co-operative society? 79 IPCC - Income Tax 18 September 2009

16) Solution
Any distribution of capital asset by a firm/ AOP/ BOI (other than company or co-operative society) on dissolution or otherwise is exigible to capital gains in the hands of such firm/ AOP/ BOI. House Listed Govt.
property Full value of consideration (FMV as on the date of transfer) Less: Cost of acquisition (1,00,000 x 582 100) Long-term capital gains Short-term capital gains 800,000 582,000 218,000 securities 600,000 500,000
100,000

18 September 2009

IPCC - Income Tax

80

CAPITAL GAINS ON DISTRIDUTION OF ASSETS BY COMPANY IN LIQUIDATION [SEC.46] (1) In hands of company: distribution of assets by a company on its liquidation is not regarded as transfer. (2) in the hands of shareholder: Receipts of any money or other assets by the shareholder from the company on its liquidation shall be chargeable to tax as FollowsCash received or market value of the assets received on liquidation Less: deemed dividend u/s 2(22) (c) to the extent of accumulated profit as on the date of liquidation. Full value of consideration for the purposes of section 48. Less: indexed cost of acquisition (or cost of acquisition) of the shares held in that company Long-term capital gains or short-term capital gains XX XX ---XX XX

XX

(3) C.O.A.of assets received on liquidation in hands of shareholders [Sec.55 (2)]: Where any capital received by assessee on liquidation of a company, which had been assessed u/s 46, is transferred by him, the cost of acquisition in of such asset will be the fair market value as on the date of distribution.
18 September 2009 IPCC - Income Tax 81

18) Question
Shri Aniket purchased 1,00,000 shares of Nahar Spinning mills Ltd. (50 % of total shares of the company) in 1992-93 for Rs. 8,50,000. The company was liquidated on 17-12-2008 and on liquidation he received Rs. 15 per share and immoveable property whose market value worth Rs. 17,50,000. On liquidation the company possessed accumulated profits of Rs. 8,00,000 (after making necessary provision for corporate dividend tax). Find out the capital gains in hands of Aniket for the assessment year 2009-10.

18 September 2009

IPCC - Income Tax

82

18) Solution
Computation of capital gains in case of Shri Aniket for the Assessment Year 2009-10
Amount received on liquidation (Rs. 15 x 1,00,000 + Rs. 17,50,000)
Less: Amount taxable as deemed dividend i.e. share in accumulated

3,250,000

profits (50% of 8,00,000) Full value of consideration


Less: Indexed cost of acquisition (Rs. 8,50,000 x 582 + 223) Long term capital gains
18 September 2009 IPCC - Income Tax

400,000 2,850,000 2,218,386 631,614


83

Q19)Ms. Vasumathi purchased 10,000 equity shares of Rajesh Co. Pvt. Ltd. on 28.2.2004 for Rs. 1,20,000. The company was wound up on 31.7.2008. The following is the summarized financial position of the company as on 31.7.2008 : Liabilities 60,000 Equity shares General reserve Provisions for taxation Rs. Assets Rs. 4200,000 650,000

600,000 Agricultural lands 4000,000 Cash at bank 250,000 4850,000

4850,000

The tax liability (towards dividend distribution tax) was ascertained at Rs. 3,00,000, after considering refund due to the company. The remaining assets were distributed to the shareholders in the proportion of their shareholding. The market value of 6 acres of agricultural land (in an urban area) as on 31.7.2008 is Rs. 10,00,000 per acre. The agricultural land received above was sold by Ms. Vasumathi on 29.2.2009 for Rs. 15,00,000. Discuss the tax consequences in the hands of the company and Ms. Vasumathi. Cost inflation indices for financial year 2003-04 is 463 and for financial year 2008-09 is 582. (CA PCC May 2008) (8 Marks)
18 September 2009 IPCC - Income Tax 84

19) Solution- (A)


Tax Consequences in the hands of Company : The net tax liability towards dividend distribution tax is ascertained at Rs. 3,00,000 which shall be deducted while computing accumulated profits deemed as dividend u/s 2(22)(c). It is assumed that the liability towards dividend distribution tax includes the amount of provision for taxation given in the balance sheet. Therefore, only the additional provision of Rs. 50,000 will be deducted and the accumulated profits Balance in General Reserve A/c of the company shall be computed as follows 4,000,000
Less : Provision for dividend distribution tax Accumulated profits or total amount to be deemed as dividend u/s 2(22)(c) 50,000 3,950,000

The company has to pay dividend distribution tax on the aforesaid amount of deemed dividend, which is already given in the question i.e. Rs. 3 lakhs. Total distribution on liquidation = Market value of assets i.e. agricultural lands + Cash balance after deducting the sum requisite for taxes = 60 lakhs + 6,50,000 - 2,50,000 (for amount of taxes given in the question) - 50,000 (for additional dividend distribution = Rs. 85 IPCC tax) - Income Tax 63.5 lakhs. 18 September 2009

(B) Computation of Capital Gains of Ms. Vasumathi for the assessment year 2009-10 Rs.)
Capital gains on assets received at the time of liquidation of Company Proportionate market value of the assets received on liquidation [63.5 lakhs x 10,000 + 60,000] Less : Deemed dividend under Section 2(22)(c) [39.5 lakhs + 6] . Full value of consideration for the purposes of section 48 Less : Indexed cost of acquisition [1,20,000 x 582 + 463] Long-term Capital Gains Capital Gain on sale of urban agricultural land : value of urban agricultural land Less : Cost of acquisition (being market value taken for computing capital gains on
liquidation)

(in
Amount 1,058,333 658,333 400,000 150,842 249,158

1,500,000 1,000,000 500,000

Short term Capital Gains

18 September 2009

IPCC - Income Tax

86

CAPITAL GAINS ON BUY-BACK OF SHARES OR OTHER SPECIFIED SECURITIES [Section 46A]


Any consideration received by a holder of shares or other specified securities from any company under a scheme of buy back shall constitute transfer and the difference between such consideration and the cost (or indexed cost) of acquisition shall be chargeable to tax as capital gains in the previous year in which such buy-back takes place. Payment made by a company on buy-back doesnt constitute dividend u/s 2(22) (d).
18 September 2009 IPCC - Income Tax 87

CAPITAL GAINS IN CASE OF DEPRECIABLE ASSETS [Section 50 & 50A]


1. Capital gains in case of transfer of asset on which depreciation has been allowed under Section 32(1)(ii) in respect of block of assets [Section 50] : The capital gains shall be computed as follows : (a) Block of assets does not cease to exist but WDV of block is reduced to zero [Section 50(1)]:

Full value of consideration Less : (1) Expenses on transfer (2)WDV of asset on 1st day of the previous year (3)Cost of assets acquired during the previous year and falling within that block Short Term Capital Gains
18 September 2009 IPCC - Income Tax

XXX XXX XXX XXX

XXX
88

CAPITAL GAINS IN CASE OF DEPRECIABLE ASSETS [Section 50 & 50A]


(b) Block of assets ceases to exist due to the sale of all assets falling within that block [Section 50(2)]:
Full value of consideration Less : (1) Expenses on transfer (2) WDV of asset on 1st day of the previous year (3) Cost of assets acquired during the previous year and falling within that block Short Term Capital Gains/Loss XXX XXX XXX XXX

XXX

18 September 2009

IPCC - Income Tax

89

CAPITAL GAINS IN CASE OF DEPRECIABLE ASSETS [Section 50 & 50A]


Transfer of capital assets of Power sector units on which depreciation allowed u/s 32(1) (i) [Section 50A]: (a) If WDV of the asset exceeds Moneys Payable on transfer of such assets: Terminal depreciation under Section 32(1) (iii) = WDV of such asset Moneys Payable (b) If Moneys Payable exceeds WDV of the asset: Then, if Moneys payable doesnt exceed actual cost : Balancing charge u/s 41(2) = Money Payable WDV Moneys payable exceeds Actual Cost : Balancing Charge u/s 41(2) = Actual Cost WDV; and Short-term/Long-term Capital Gains = Moneys Payable Actual Cost

18 September 2009

IPCC - Income Tax

90

20) Question
The written down value of the block of assets of Rosy Ltd. as on 1st April, 2008 was Rs.5 lakh. An asset of the same block was acquired on 11th May, 2008 for Rs.3 lakh. There was a fire on 18th September, 2008 and the assets were destroyed by fire and the assessee received a sum of Rs.11 lakh from the insurance company. Compute the capital gain assuming All the assets were destroyed by fire; and 1. Part of the block of assets was destroyed by fire. What will be the answer if assessee received Rs.6 lakh from insurance company instead of Rs.11 lakh ? 91 IPCC - Income Tax (CS Dec. 18 September 2009 2006)

20) Solution
Compensation received is Rs. 11 lakh : Computation of Capital gains
Written down value of the block on 1/4/08 Add: Asset acquired during the year
Less: Sum received from the insurance company Short term capital Gains

500,000 300,000 800,000 1,100,000 300,000

Note : In case (i) and (ii), both, i.e. whether the block is fully destroyed or partly destroyed by fire, there will be STCG of Rs. 3,00,000, since the sum received is more than the WDV of the block. If compensation received is Rs. 6 lakh : 1. If the block is fully destroyed : The difference between WDV of Rs. 8 lakh and insurance money of Rs. 6 lakh will be short-term capital loss. 2. If the block is partly destroyed : There will be no capital gains. Since the block and WDV both exist, therefore, the balance WDV of Rs. 2 lakh will be eligible for depreciation.
18 September 2009 IPCC - Income Tax

92

SLUMP SALE MEANING AND COMPUTATION OF CAPITAL GAINS [Section 50B]


Slump Sale [Sec. 2(42C)] : It means transfer of one or more undertakings as a result of the sale for a lump sum consideration without values being assigned to the individual assets and liabilities in such sales. Charge and Nature of Capital Gains: Profits or gains arising from slump sale shall be taxable as Capital Gains in previous year in which slump sale is effected. If the capital asset, being one or more undertakings, was owned and held by the assessee for not more than 36 months, the capital gains will be short term capital gains. In any other case, it shall result into long-term capital gains. Mode of computation of capital gains: The capital gains shall be computed in the following manner Full value of consideration Less : Expenses wholly and exclusively in connection with such transfer Less : Cost of acquisition and cost of improvement being net worth** of the undertaking (no indexation benefit even in case of long-term capital asset) Short Term/Long Term Capital Gains
18 September 2009 IPCC - Income Tax

XXX XXX XXX

XXX
93

SLUMP SALE MEANING AND COMPUTATION OF CAPITAL GAINS [Section 50B]


SLUMP SALE MEANING AND COMPUTATION OF CAPITAL GAINS [Section 50B]

Aggregate value of total assets of the undertaking or division (ignoring any change in the value of assets on account of revaluation of assets) In case of depreciable assets, the WDV of the block as per Sec. 43(6) XXX In case of other assets, the book value XXX Less : Value of liabilities of such undertaking or division as appearing in its books

XXX

XXX XXX

Net Worth of the undertaking or division

XXX

18 September 2009

IPCC - Income Tax

94

SLUMP SALE MEANING AND COMPUTATION OF CAPITAL GAINS [Section 50B]

Certificate of a Chartered Accountant: In case of slump sale, every assessee shall furnish along with the return of income a report of an accountant in prescribed form indicating the computation of net worth and certifying that the net worth of the undertaking or division has been correctly arrived at.
18 September 2009 IPCC - Income Tax 95

Illustration 1:
Computation of Capital gains in case of slump sale Balance Sheet of X Ltd. as on December 31, 2006 reads as under Paid up capital: Rs.552 lakhs (All amounts in Rs. lakhs)
Unit A Unit B Land Fixed Assets (other than land) Debtors Liabilities Stock-in-trade Reserves Share Premium Revaluation Reserve on account of Revaluation of land
18 September 2009 IPCC - Income Tax

200 100 100 28 50

170 150 75 50 25 148 22 70


96

Illustration 1:
The company acquired Unit B on December 31, 2003. It made certain capital additions in the form of generator set and additional building, etc. of Rs.25 lakhs during the year 2004-05. The members of the company have authorized the Board in their meeting held on October 28, 2006 to dispose off the Unit B. The company decides to sell the Unit B by way of slump sale for Rs.325 lakhs as consideration. The buyer has agreed with the vendor company to give time for putting through the sale but not later than March 31, 2007 subject to discount of 1% on agreed sale consideration. However, this discount is not applicable if the sale is completed after December 31, 2006. The company now approaches you to advise them as a measure of tax planning to determine the date of sale keeping in view of the capital gains tax. The WDV of the Fixed Assets under Section 43(6) is Rs.120 lakhs.

18 September 2009

IPCC - Income Tax

97

Solution:
The computation of capital gains shall be in accordance with Section 50B of the Income Tax Act, 1961. Computation of net worth of Unit B: Rs. (in lakhs)

WDV of Fixed Assets under Section 43(6) being depreciable assets Land (170 70 i.e. book value after ignoring revaluation) Debtors and Stock (75 + 25 i.e. at book value) Less : Liabilities at Book Value Net Worth (being cost of acquisition and improvement)
18 September 2009 IPCC - Income Tax

120 100 100

320

50 270
98

Solution:
Case I: Where the slump sale takes place on or before December 31, 2006 In this case, the period of holding of Unit B will be not more than 36 months. Therefore, Unit B will be a short-term capital asset. Sale Consideration Less : Discount @ 1% Less : Net Worth 325.00 3.25 270.00

Short Term Capital Gains Tax payable (30% + Surcharge 10% + EC @ 2% on Tax and Surcharge)
18 September 2009 IPCC - Income Tax

51.75 17.42
99

Solution:
Case II : Where Unit B is sold after December 31, 2006 : In this case, period of holding is for more than 36 months and therefore, it is a long-term capital asset.

Sale Consideration Less : Net Worth Long Term Capital Gains Tax payable = 20% + Surcharge 10% + EC @ 2% on Tax and Surcharge

325 270 55 12.34

Conclusion: Since the tax payable is less in Case II, X Ltd. is advised to sell Unit B by way of slump sale after December 31, 2006 so as to minimize the tax liability.
18 September 2009 IPCC - Income Tax 100

FULL VALUE OF CONSIDERATION WHEN STAMP VALUE EXCEEDS SALE PRICE [Section 50C]
Full Value of Consideration : Where the consideration for transfer of land or building or both, is less than the value adopted by Stamp Valuation Authority for payment of stamp duty, the value so adopted by stamp valuation authority shall be deemed to be full value of consideration for the purpose of Section 48. Reference to Valuation Officer: The Assessing Officer may refer valuation thereof to Valuation Officer if The assessee claims before the Assessing Officer that the value adopted or assessed by the Stamp Valuation Authority exceeds the fair market value of the property as on the date of transfer, and The value adopted or assessed by the Stamp Valuation Authority has not been disputed in any appeal or revision or no reference has been made before any other authority, court or the High Court. In case reference is made to Valuation Officer, the full value of consideration shall be lower of (a) Value as determined by the Valuation Officer; or (b) Value assessed or adopted by the Stamp Valuation Authority.

18 September 2009

IPCC - Income Tax

101

21)Question
Bala sold his vacant site on 31.09.2008 for Rs. 7,00,000. It was acquired by him on 01.10.1997 for Rs. 1,50,000. The State stamp valuation authority fixed the value of the site at the time of transfer at Rs. 13,00,000. Compute capital gains in the hands of Bala and your reasons for computation. (May 2004)
18 September 2009 IPCC - Income Tax 102

21)Computation of capital gains of Bala for the assessment year 2009-10 (Amt. in Rs.)

Full value of consideration (Higher of (a) Stamp value Rs. 13 lakh; or (b) proceeds Rs. 7 lakh) Less: Indexed cost of acquisition (1,50,000 x 582 331)
Long-term capital gain

1,300,000 263,746
1,036,254

18 September 2009

IPCC - Income Tax

103

CAPITAL GAINS WHEN ADVANCE OR OTHER MONEY FORFEITED [Section 51] Where any capital asset was on any previous occasion the subject of negotiations for its transfer, and advance or other money received and retained by the assessee in respect of such negotiation shall be deducted from the cost for which the asset was acquired, or the WDV of the asset or the FMV in computing the cost of acquisition of the capital asset. [Note: Only amount forfeited by assessee is deducted, amount forfeited by the previous owner shall not be considered. Further, indexation applies only after such reduction from cost.] It has been held in Travancore Rubber & Tea Co. Ltd. v. CIT [2000] 243 ITR 158 (SC) that the phrase other money would cover deposits made by purchaser for guaranteeing due performance of contracts. Therefore, forfeiture of earnest money and the compensation awarded to the assessee for breach by the prospective purchaser of contract for purchase of property would go to reduce the cost of acquisition as per Section 51.

18 September 2009

IPCC - Income Tax

104

22) Question
A house was purchased on 1.5.1995 for Rs. 2 lakhs and was used as a residence by the owner. The owner had contracted to sell this property in June 2005 for Rs. 8 lacs and had received an advance of Rs. 50,000 towards sale. The intending purchaser did not proceed with the transaction and the advance was forfeited by the owner. The property was sold in July 2008 for Rs. 10,00,000. The owner, from out of sale proceeds, invested Rs. 3 lacs in new residence in December 2008. Compute the net taxable capital gains. (May 1993)
18 September 2009 IPCC - Income Tax 105

22) Solution Computation of Long Term Capital Gains for the Assessment Year 2009-10 (amounts in Rs.)
Sale Consideration Less: Indexed Cost of Acquisition (1,50,000 x 582 281) Long Term Capital Gain Less: Exemption u/s 54 (Cost of purchase of new residential house property) Taxable Long Term Capital Gain 1,000,000 310,676 689,324

300,000 389,324

Note : As per section 51, any advance money or any other sum received and retained by the assessee (forfeited by the assessee) shall be reduced from the cost of acquisition. Hence, indexed cost of acquisition is calculated only on Rs. 1,50,000 (i.e. Rs.2,00,000 - Rs.50,000).
18 September 2009 IPCC - Income Tax 106

Illustration 2 Right to specific performance:

R entered into an agreement with L for the purchase of a property for Rs.10 lakhs and paid Rs.10, 000 as earnest money. On L failing to execute a conveyance in respect of the property, a suit for specific performance was filed by R. The suit was compromised and R agreed to receive Rs.50, 000 by way of damages and gave up his right to specific performance. What will be the position of this amount?
18 September 2009 IPCC - Income Tax 107

Solution Right to specific performance It was held in K.R. Srinath v. ACIT [2004] 268 ITR 436 (Mad.) that a right to acquire a property i.e. right to specific performance is itself a capital asset. In this case, Rs.50, 000 received by R is consideration for relinquishment of right to specific performance and Rs.10, 000 is its cost of acquisition. R is therefore, liable to capital gain tax on Rs.40, 000 subject to deduction of the legal expenses incurred for enforcing this right.

18 September 2009

IPCC - Income Tax

108

EXEMPTIONS IN RESPECT OF CAPITAL GAINS AVAILABLE ONLY TO INDIVIDUAL AND/OR HUF ASSESSEES [Section 54, 54B and 54F]
Provisions 1. Assessee 2. Asset transferred Section 54 Individual/HUF Residential house property being buildings or lands appurtenant thereto. Section 54B Individual Agricultural land used by individual or his parent for agricultural purposes during 2 years preceding date of transfer. Section 54F Individual/HUF Any capital asset not being residential house property. [Note : Exemption is not available if assessee (a) owns more than 1 residential house (other than new) on date of transfer of original asset; or (b) purchases a residential house, other than new asset, within 1 year from date of transfer of original asset]

3. Nature of Asset 4. New asset to be purchased/ constructed 5. Time-limit for purchase/ construction

Long Term Residential house property i.e. buildings or lands appurtenant thereto Purchase : Within 1 year before or 2 years after the date of transfer Construction : Within 3 years from date of transfer

Short/Long Term Agricultural land (urban or rural) Purchase within 2 years from the date of transfer

Long Term Residential house property i.e. buildings or lands appurtenant thereto Purchase : Within 1 year before or 2 years after date of transfer; and Construction : Within 3 years from date of transfer

18 September 2009

IPCC - Income Tax

109

EXEMPTIONS IN RESPECT OF CAPITAL GAINS AVAILABLE ONLY TO INDIVIDUAL AND/OR HUF ASSESSEES [Section 54, 54B and 54F]
6. Deposit Scheme (discussed later) 7. Amount of Exemption Applicable Applicable Applicable

Lower of Capital Gains or Investment in new asset

Lower of Capital gains or cost of new asset

Cost of new house Capital Gains Net consideration being Full value of consideration less Expenses on transfer (a) assessee purchases within 2 years or constructs within 3 years from date of transfer of original asset, a residential house other than new house; or (b) Transfers new asset within 3 years from date of its purchase/ construction. Amount exempted earlier shall be taxable as longterm capital gains in previous year in which (a) another residential house is purchased or constructed; or (b) the new asset is transferred.

8. Withdrawal of exemption on

Transfer of the new asset within 3 years from its purchase/ construction

Transfer of the new asset within 3 years from its purchase

9. Taxability on withdrawal

Amount of exemption claimed earlier shall be reduced from the cost of acquisition of new asset

Exemption claimed earlier shall be reduced from cost of acquisition of new asset

18 September 2009

IPCC - Income Tax

110

EXEMPTIONS IN RESPECT OF CAPITAL GAINS AVAILABLE ONLY TO INDIVIDUAL AND/OR HUF ASSESSEES [Section 54, 54B and 54F] Note: Important points on exemption under Section 54 and 54F Purchase/Construction of a Portion: Purchase or consideration of a portion of the house is eligible for exemption CIT v. Chandanben Maganlal [2000] 245 ITR 182 (Guj.). E.g. If an assessee purchases 15% undivided share in a house property, exemption will be available. However, mere construction by way of extension of old existing house is not eligible for exemption. CIT v. Pradeep Kumar [2006] 153 Taxman 138 (Mad.) Purchase of co-owners interest : In case of property owned by co-owners, the payment made by one co-owner to get the full ownership by release of the interest of other co-owners amounts to purchase by such co-owner and is eligible for exemption. CIT v. Aravinda Reddy [1979] 120 ITR 46 (SC). Registration not pre-condition: If assessee has purchased house and acquired its possession and control, he will be eligible for exemption even if such purchase is not registered as per Registration Act, 1908.

18 September 2009

IPCC - Income Tax

111

EXEMPTIONS IN RESPECT OF CAPITAL GAINS AVAILABLE TO ALL ASSESSEES [Section 54D, 54EC, 54G and 54GA]:
Provisions 1. Assessee 2. Asset transferred Section 54D Any person Compulsory acquisition of land or building which was used in the business of industrial undertaking during 2 years prior to date of transfer. Short term/ term Long Section 54EC Any person Any long term capital asset. Section 54G Any person Transfer of plant, machinery or land or building for shifting industrial undertaking from urban area to rural area. Section 54GA Any person Transfer of plant, machinery or land or building for shifting industrial undertaking from urban area to Special Economic Zone. Short term/ term (a) Long

3. Nature of Asset 4. New asset to be purchased/ constructed

Long term Bonds, redeemable after 3 years issued (a) by National Highway Authority of India; or (b) By Rural Electrification Corp. (Amendment by the Finance Act, 2006)

Short term/ term (a)

Long

New land or building for the industrial undertaking

(b)

(c)

Purchase/ Construction of plant, machinery, land or building in such rural area or, Shifting original assets to that area, or Incurring notified expenses

(b)

(c)

Purchase/ construction of plant, machinery, land or building in such SEZ, or Shifting the original assets to SEZ, or Incurring notified expenses

18 September 2009

IPCC - Income Tax

112

EXEMPTIONS IN RESPECT OF CAPITAL GAINS AVAILABLE TO ALL ASSESSEES [Section 54D, 54EC, 54G and 54GA]:
6. Deposit Scheme 7. Amount of exemption Applicable Lower of capital gains or investment in new asset Transfer of new asset within a period of 3 years from the date of its acquisition or construction -Lower of Capital gains or investment in new asset or Rs.50 lacs Transfer of new asset, conversion thereof in money or taking loan or advance on its security within 3 years from date of its acquisition. Exempted capital gain will be taxable as longterm capital gains in previous year in which such transfer/ conversion takes place. Applicable Lower of Capital gains or cost incurred for (a) to (c) of point 4. Transfer of new or shifted asset within a period of 3 years from the date of its acquisition or construction or shifting. Applicable Lower of Capital gains or cost incurred for (a) to (c) of point 4. Transfer of new or shifted asset within a period of 3 years from the date of its acquisition or construction or shifting.

8. Withdrawal of Exemption

9. Taxability on withdrawal of exemption

Amount of exemption claimed earlier shall be reduced from the cost of acquisition of new asset.

Amount of exemption claimed earlier shall be reduced from the cost of acquisition of new or shifted asset.

Amount of exemption claimed earlier shall be reduced from the cost of acquisition of new or shifted asset.

Note: If exemption has been claimed u/s 54EC in respect of investment in a new asset, no deduction shall be allowed u/s 80C with reference to the amount of investment for which exemption has been claimed.

18 September 2009

IPCC - Income Tax

113

EXEMPTIONS IN RESPECT OF CAPITAL GAINS AVAILABLE TO ALL ASSESSEES [Section 54D, 54EC, 54G and 54GA]:
Transfer of depreciable assets held for more than 36 months Exemption u/s 54EC available: Section 50 nowhere mentions that the depreciable assets are short term capital assets but only states that capital gains arising from transfer of depreciable asset shall be deemed to be arising out of transfer of short term capital asset. Section 54EC is independent section and exemption therein is available if there is a transfer of long term capital asset and consideration is invested in specified assets within time limit. Therefore, depreciable assets held for more than 36 months are long-term capital assets and capital gains arising therefrom will be eligible for the benefit envisaged u/s 54EC CIT v. Assam Petroleum Industries P. Ltd. [2003] 131 Taxman 699 (Gau.) Extension of time in case of compulsory acquisition [Section 54H] : Where transfer of original assets referred to in Sections 54, 54B, 54D, 54EC and 54F, is by way of compulsory acquisition under any law, the period for acquiring new asset referred to in those sections or the period available under those sections for depositing or investing the amount of capital gain in relation to such compensation, which is not received on the date of the transfer, shall be reckoned from the date of receipt of such compensation.
18 September 2009 IPCC - Income Tax 114

EXEMPTIONS IN RESPECT OF CAPITAL GAINS AVAILABLE TO ALL ASSESSEES [Section 54D, 54EC, 54G and 54GA]:
Capital Gains Accounts Scheme, 1988: This scheme applies to all assessees who are eligible for exemption under Section 54, 54B, 54D, 54F and 54G. The tax implications of this scheme are as follows
Exemption available if amount deposited: Exemptions u/s 54, 54B, 54D, 54F and 54G are available if the investment in new asset is made within time allowed in those sections. If the amount of capital gains or net consideration could not be fully or partly reinvested for the purposes specified in said sections before the due date of furnishing return of income, then exemption will be available in respect of the amount deposited before the due date of furnishing return of income in the said deposit account as if the amount so deposited had been invested in new asset. Withdrawal out of deposit account: The amount in deposit account can be withdrawn for purposes specified in respective Sections 54, 54B, 54D, 54F and 54G. However, if the said amount is not utilized wholly or partly for purchase of new asset within stipulated period specified under said sections, then

In case exemption was claimed u/s 54, 54B, 54D and 54G : Amount not so utilized shall be chargeable to tax as Capital Gains of previous year in which period specified under those section expires. In case exemption was claimed under Section 54F : The following amount shall be taxable as capital gains of previous year in which the period under Section 54F expires
18 September 2009 IPCC - Income Tax 115

EXEMPTIONS IN RESPECT OF CAPITAL GAINS AVAILABLE TO ALL ASSESSEES [Section 54D, 54EC, 54G and 54GA]:

Taxable Amountnot so utilised OriginalCapitalGains(beforeclaimingexemption) Capital = Net sale considerat ion in respectof transferof originalasset Gains

If the individual dies before expiry of stipulated period u/s 54, 54B, 54D, 54F and 54F, the unutilized amount cannot be taxed in the hands of the deceased, also not in hands of legal heirs, as the unutilized portion is not income but is only a part of the estate devolving upon them. (Circular 743 dt. 06.05.1996)

18 September 2009

IPCC - Income Tax

116

Illustration 3 Exemption u/s 54 and 54F:


Mr. A owns a self-occupied residential house and a plot of land. (He has no other house). He sells the house on 31.1.2007 and the plot on 15.2.2007 for Rs.6, 50,000 and Rs.5, 00,000 respectively. The house was purchased on 31.1.2002 for Rs.4, 00,000 and the plot on 30.3.2002 for Rs.2, 00,000. A has purchased a new residential house on 25.4.2007 for Rs.5, 00,000 and claims exemption in respect of such house. On 31.1.2008, he transfers the said residential house for Rs.7, 50,000 and purchases a new house on 31.3.2008 for Rs.10, 00,000. Compute the capital gains for relevant years.

18 September 2009

IPCC - Income Tax

117

Solution Exemption u/s 54 and 54F:


Computation of Capital Gains for assessment year 2007-08

Sale of residential house (Rs.)

Sale of Plot (Rs.)

Full value of consideration Less : Indexed cost of acquisition Long term capital gains Less : Exemption u/s 54 & 54F Taxable Capital Gains

6,50,000 4,87,324 (4,00,000 519/426) 1,62,676 1,62,676 Nil

5,00,000 2,43,662 (2,00,000 519/426) 2,56,338 1,72,938 (2,56,338 3,37,324 5,00,000) 83,400

18 September 2009

IPCC - Income Tax

118

Solution Exemption u/s 54 and 54F:


Computation of Capital Gains on sale of residential house (amount in Rs.)
Sale price of the residential house (acquired on 25.4.2007) Less : Cost of Acquisition (5,00,000 Exemption claimed u/s 54 i.e. 1,62,676) 7,50,000 3,37,324

Short-term Capital Gains for assessment year 2008-09

4,12,676

Long-term Capital Gains (Exemption claimed u/s 54F shall be chargeable as long-term capital gains of the year in which the house is transferred i.e. assessment year 2007-08)

1,72,938

Note: No exemption will be available in respect of second new house acquired on 31.3.2008. Exemption u/s 54 or 54F cannot be claimed because the house transferred on 31.1.2008 is a short-term capital asset.
18 September 2009 IPCC - Income Tax 119

23) Question
Mr. Surinder furnishes the following particulars for the previous year ending 31-3-2009, and requests you to compute the taxable capital gain : 1. He had a residential house inherited from father in 1996, the fair market value of which as on 1-4-1981 is Rs. 5,00,000. 2. In the year 1998-99, further construction and improvements costed Rs. 6 lakhs. 3. On 10-5-2008 the house was sold for Rs. 50 lakhs. Expenditure in connection with transfer Rs. 50,000. On 20-12-2008, he purchased a residential house for Rs. IPCC - Income Tax 18 September 2009 15 lakhs. (May 2002) 120

23) Solution Computation of Capital Gains (amounts in Rs.) Consideration Received


Less: Expenses on Transfer Net Consideration Less: Indexed Cost of Acquisition (Rs. 5,00,000 x 582 305) (Note) Less: Indexed Cost of Improvement (Rs. 6,00,000 x 582 351) Long Term Capital Gains Less: Exemption u/s 54 being the lower of Long Term Capital Gain Cost of new house acquired Taxable Long Term Capital Gains 954098 994872

5000000 50000 4950000

1948970 3001030

3001030 1500000

1500000 1501030

Note : Since Mr. Surinder inherited the property, which is a mode of acquisition specified in section 49(1), therefore, the cost shall be the cost of acquisition of the previous owner, which, in this case, is Rs. 5 lakhs. However, for indexation purposes, the Cost Inflation index of that year will be taken in which the asset was first held by Mr. Surinder i.e. CII of the year of inheritance being 1996-97 will be 121 IPCC - Income Tax 18 September 2009 taken.

24) Question
Raghavan owned a residential house at Madurai, the original cost of which was Rs. 1,00,000. It was acquired on 1-9-1999. He sold the house on 1-6-2004 for Rs. 8,00,000 and purchased another house on 31-5-2006 at Tiruchi for Rs. 6,00,000. The second house at Tiruchi was sold by him for Rs. 8,00,000 on 306-2008. Discuss the impact of these transactions with regard to assessment to capital gains. (May 2000)
18 September 2009 IPCC - Income Tax 122

24) Solution:- (1) Computation of Long Term Capital Gain for the assessment year 2005-06 (amounts in Rs.)
Sale value Less: Indexed cost of acquisition (1,00,000 x 480 389) 123,393 Long-term capital gains 676,607 Less: Exemption u/s 54 being lower of : 676,607 Long Term Capital Gain Cost of New Property 600,000 600,000 Taxable capital gain 76,607 800,000

18 September 2009

IPCC - Income Tax

123

24) Solution:- (2)


Short Term Capital Gains for the assessment year 2009-10 on sale of house = Full value of consideration - Cost of acquisition* = 8,00,000 NIL = Rs. 8,00,000. *Since exemption was claimed u/s 54 in respect of the house and the same has been sold within 3 years from the date of its acquisition, therefore, the exemption will be 'withdrawn. Accordingly, cost of acquisition of house = Original Cost of purchase - Exemption claimed u/s 54 = 6 lakh 124 IPCC - Income Tax 18 September 2009 6 lakh = Nil.

25) Question
S, an owner of 3 houses, sells a residential house property in Chennai for Rs. 10,90,000 on 23-5-2008. This house was purchased by him in April, 1994 for Rs. 2,80,000. On 30-5-2008, he purchased a flat in Mumbai of Rs. 8,70,000 for the purpose of the residence of his son-in-law. On 1-3-2009 S sells the flat in Mumbai for Rs. 12,10,000. Compute the capital gains arising on the two transactions. Is S eligible for exemption under section 54 in respect of the second sale? 125 IPCC - Income Tax 18 September 20091994) (Nov.

25) Solution Computation of capital gains of Mr. S for the assessment year 2009-10 (amounts in Rs.)
proceeds of the property located in Chennai Less : Indexed Cost of Acquisition (Rs. 2,80,000 x 582 259) Long Term Capital Gain (1) Sales Consideration from the flat Less : Cost of Acquisition Short Term Capital Gain (2) Total Capital Gains (1 + 2) 1,090,000 629,189 460,811 1,210,000 870,000 340,000 800,811

Note : In this case, after transfer of residential house, Mr. S has purchased a new flat during the previous year 2008-09 and sold the same in the previous year 2008-09 itself. If Mr. S claims exemption in respect of the flat while computing long-term capital gains, then, on sale of the flat, the cost of the flat will be reduced by the amount of exemption claimed. This will have the effect of decreasing the amount of long-term capital gains and increasing the amount of short-term capital gains. While long-term capital gains are taxable @ 20%; short-term capital gains are taxable as per normal rates (highest slab being 30%). Therefore, in order to avoid higher tax liability, Mr. S should not claim exemption in respect of the flat. Further, no exemption is available in respect of sale of new house, as the same 126 IPCC - Income Tax 18 2009 capital asset. is September a short-term

26) Question
Ms. Vimla sold a residential building at Jodhpur for Rs. 15,00,000 on 1-7-2008. The building was acquired for Rs. 1,50,000 on 1-6-1996. She paid brokerage @ 2% at the time of sale of the building. She invested Rs. 7 lakhs in purchase of a residential building in December, 2008 and deposited Rs. 2 lakhs in NHAI Capital Gains Bond in March, 2009. Compute her taxable capital gains for the assessment year 2009-10. (Nov. 2003)
18 September 2009 IPCC - Income Tax 127

26) Solution : Computation of Capital Gains for A.Y. 2009-10


(All amounts in Rs.)

Full value of consideration


Less: Indexed cost of acquisition [1,50,000 x 582 Less: Expenses on transfer @ 2% of Rs. 15 lakh

1,500,000 305] 286,230 30,000 1,183,770 700,000


483,770

Long term Capital Gain Less: Exemption under section 54 Taxable Long term capital gains

18 September 2009

Note: Benefit under section 54EC shall not be available since investments are made after 6 months from the date of transfer of capital asset. IPCC - Income Tax

128

27) Question
Arjun furnishes the following particulars and requests your advise as to the liability in capital gains for the assessment year 2009-10 : 1. Jewellery purchased by him on 10-3-2000, for Rs. 1,05,000 was sold by him for a consideration of Rs. 2,85,000 on 2-11-2008. 2. He incurred expenses : a. At the time of purchase Rs. 2,000 b. At the time of sale (for brokerage) Rs. 4,000 3. He invested Rs. 70,000 in bonds with National Highway Authority of India out of sale consideration. On these facts : a. Compute the capital gains chargeable to tax. b. Whether Arjun would be entitled to any exemption? (Nov. 2001)

18 September 2009

IPCC - Income Tax

129

27) Solution Computation of capital gain for the assessment year 2009-10 (amounts in Rs.)
Sale consideration Less : Expenses on Transfer Net Sale Consideration Less : Indexed cost of acquisition (Rs. 1,07,000 x 582 389) Long-term capital gain Less: Exemption u/s 54EC being the lower of Long-term capital gain Investment in bonds of NHAI Taxable long-term capital gain 285,000 4,000 281000 160087

120913

120913 70000 70000 50913

18 September 2009

IPCC - Income Tax

130

28) Question
Mr. 'X' furnishes the following data for the previous year ending 31.3.2009 : (a) Equity Shares of AB Ltd., 10,000 in number were sold on 31.5.2008, at Rs. 350 for each share. (b) The above shares of 10,000, were acquired by 'X' in the following manner: Received as gift from his father on 1.6.1980 (5,000 shares) the market price on 1.4.81 Rs.50 per share. Bonus shares received from AB Ltd. on 21.7.1984 (2,000 shares). Purchased on 1.2.1993 at the price of Rs.125 per share (3,000 shares). (c) Purchased one residential house at Rs.25 lakhs, on 1.9.2009 from the sale proceeds of shares. (d) 'X' is already owning a residential house, even before the purchase of above house. You are required to compute the taxable capital gain. He has no other source of income chargeable to tax. (Cost Inflation Index - Financial year 1992-93: 223; Financial year 2008-2009 : 582) (Nov. 2004)(6 Marks)
18 September 2009 IPCC - Income Tax 131

28) Solution : Computation of taxable capital gain of Mr. 'X' for A.Y. 2009-10 (amounts in Rs.)
Particulars Gifted shares (5000)* (A) 1750000 1455000 295,000 16.86%
II

Purchased shares (3000)** 1050000 978,700 71,300


6.79% III

Bonus Shares (2000) 700000 0 700000


100.00% I

Sale consideration @ Rs. 350 per shares


& (**3000 x125 x582 223)

Less : Indexed cost of acquisition (*5000 x 50 x 582 + 100) Long Term Capital Gains (B) % of LTCG to sale consideration (B A x 100)
(C) Priority for the purpose of exemption u/s 54F

Apportionment of Cost of new house as per priority towards net consideration of various capital assets sold
(D)

1750000
(E)

50000 3,395
67,905

700000 700,000

Exemption from LTCG u/s 54F (D x C)


Taxable LTCG

295,000
0

Note : (1) Cost of shares gifted by the father would be FMV as on 1-4-1981 i.e. Rs. 50 per share. Further, the cost of bonus shares alloted on or after 1-4-1981 is always NIL. (2) X owns only one house at the time of the transfer of the shares and the purchase of the new house takes places within 2 years from the date of transfer of the shares, therefore, he is eligible for exemption u/s 54F. It is assumed that Mr. X has complied with all the procedures like deposit of 132 IPCC -by Income 18 under September 2009 money the capital gains account scheme the Tax due date, as may be applicable t him.

29) Question
Mr. A owns a residential house, which is self-occupied, and also a plot of land (He has no other house). He sells the house on January 31, 2009 and the plot on February 15, 2009 for Rs. 6,50,000 and Rs. 6,00,000 respectively. The house was purchased on January 30, 2002 for Rs. 4,00,000 and the plot on March 30, 2002 for Rs. 2,00,000. A has purchased a new residential house on April 25, 2009 for Rs. 5,00,000 and claims exemption in respect of such house. On 31-1-2010, he transfers the said residential house for Rs. 7,50,000 and purchases a new house on 31-3-2010 for Rs. 10,00,000. Compute the capital gains for relevant years.
18 September 2009 IPCC - Income Tax 133

29) Solution
A can claim exemption u/s 54 in respect of capital gains on sale of residential house and exemption u/s 54F in respect of capital gains on sale of plot. Since the exemption u/s 54 is available in on investment of capital gains, therefore, first of all, exemption u/s 54 will be claimed. Thereafter, exemption u/s 54F, which is available on investment of net consideration, shall be claimed. The exemption u/s 54F will be available only in respect of balance cost of new house i.e. Original cost of new house Exemption u/s 54.
18 September 2009 IPCC - Income Tax 134

Computation of Capital Gains for assessment year 2009-10 (amounts in Rs.)


Residentia l house Plot

Full Value of consideration


Less: Indexed cost [(4,00,000 x 582 426); (2,00,000 x 582 426)] Long term capital gains Less: Exemption u/s 54

650,000 546,479
103,521

600,000 273,239
326,761

103,521 Exemption u/s 54F i.e.

Capital gain x[Cost of new house - Exemption u/s 54]/ Net consideration of plot.i.e. (326,761x (500000103521))/600000

215,923
18 September 2009 Taxable capital gains IPCC - Income Tax

135 110,838

Computation of capital gains on sale of residential house for assessment year 2010-11 (amounts in Rs.)
Short-term capital gains for assessment year 2010-11
353,521

Long-term capital gains (Exemption claimed u/s 54F shall be chargeable as long-term capital gains of the year in which the house is transferred i.e. assessment year 201011)

215,923

Note: No exemption will be available in respect of second new house acquired on 31-3-2010. Exemption u/s 54 or 54F cannot be claimed because the house transferred on 31-1-2010 is a short-term capital asset

18 September 2009

IPCC - Income Tax

136

30) Question
Section 54GA : Mrs. Malini Hari Shifted her industrial undertaking located in corporation
limits of Faridabad, to a Special Economic Zone (SEZ) on 1-12-2008 (Amount in Rs.) :
(a) Land : Purchased on 20-1-2002

426,000 2,200,000 820,000 1,139,000 740,000 600,000 115,000 300,000 500,000 100,000 420,000 200,000
IPCC - Income Tax 137

Sold
(b) Building [Construction completed on 14-3-2005], WDV of building as on 1-4-2008

Sold
(c) WDV of cars as on 1-4-2008

Sold for
(d) Expenses on shifting the undertaking (e) Assets acquired for the undertaking in the SEZ (on or before 25-6-2009): (i) (ii)

Land Building

(iii) Computers (iv) Car (v)

Machinery (Second hand)


18 September 2009

(vi) Furniture

50,000

There is no intention of investing in any other asset in this undertaking. Compute the exemption available under section 54GA for the assessment year 2009-10. Cost inflation indexes are : Financial year 200102 : 426; 2008-09 : 582. (CA PCE Nov. 2007) (8 marks)

18 September 2009

IPCC - Income Tax

138

30)Solution
Solution : Computation of capital gain and exemption u/s 54GA for the assessment year 2009-10
Land (Rs.) Full consideration
Less: Indexed

Building (Rs.) 1,139,000 820,000 0 319,000

Cars (Rs.)

2,200,000 582,000 1,618,000 0

600,000 740,000 0 -140,000 1,797,000

cost/cost of acquisition (4,26,000 x 582 426) Long-term capital gain Short-term capital gain/loss

Total capital gains (long-term as well as short-term)

Less: Exemption u/s 54GA for all assets purchased (including car & computer, which are plant;
second hand plant; but excluding furniture, which is ineligible for exemption) and expenses

incurred for shifting the undertaking (300000 + 500000 + 100000 + 420000 + 200000 + 115000)
18 September 2009 IPCC - Income Tax

1,635,000
139

Balance Taxable capital gains

162,000

There is no intention of investing in any other asset in this undertaking. Compute the exemption available under section 54GA for the assessment year 2009-10. Cost inflation indexes are : Financial year 2001-02 : 426; 2008-09 : 582. (CA PCE Nov. 2007) (8 marks)
18 September 2009 IPCC - Income Tax 140

Note : Exemption will be first claimed against long-term capital gains; so that the balance gains are short-term capital gains, which are taxable at normal rates, rather than long-term capital gains taxable @ 20%.

18 September 2009

IPCC - Income Tax

141

REFERENCE TO VALUATION OFFICER [Section 55A]

With a view to ascertaining the Fair Market Value of a capital asset, the Assessing Officer may refer the valuation of a capital asset to a Valuation Officer in following cases
In case the value of asset claimed by assessee accords with the estimate made by Registered Valuer: If the Assessing Officer is of the opinion that the value so claimed is less than it is Fair Market Value. In any other case : If the Assessing Officer is of the opinion that

[Fair Market Value of the asset Value claimed by the assessee] exceeds (i) Rs.25,000; or (ii) 15% of the value claimed by the assessee; or Having regard to the nature of the asset and relevant circumstances, it is necessary to make a reference to the Valuation Officer.

18 September 2009

IPCC - Income Tax

142

32) Question
Section 49(1), 50C and 55A : Mr. T inherited a house in Jaipur under will of his father in May, 2002. The house was purchased by his father in January, 1980 for Rs. 2,50,000. He invested an amount of Rs. 7,00,000 in construction of one more floor in this house in June, 2006. The house was sold by him in November, 2008 for Rs. 37,50,000. The valuation adopted by the registration authorities for charge of stamp duty was Rs. 47,25,000 which was not contested by the buyer, but as per assessee's request, the Assessing Officer made a reference to Valuation Officer. The value determined by the Valuation Officer was Rs. 47,50,000. Brokerage @ 1% of sale consideration was paid by Mr. T to Mr. S. The market value of house as on 1-4-1981 was Rs. 2,70,000. You are required to compute the amount of capital gain chargeable to tax for assessment year 2009-10 with the help of given information and by taking CII for the financial year 2008-09 as 582 and for financial year 2006-07 as 519. (CA PCE Nov. 2007) (9 marks)

18 September 2009

IPCC - Income Tax

143

32)Solution

Computation of capital gain for the assessment year 2009-10(Amount in Rs.)


4,725,000

Full value of consideration (Higher of Rs. 37,50,000 and 47,25,000; subject to maximum of Rs. 47,50,000) (See Note 1) 351,544
Less : Indexed cost (Rs. 2,70,000 x 582 447) (See Note 2)

784,971 Indexed cost of improvement (Rs. 7,00,000 x 582 519)

Expenses of transfer i.e. brokerage @ 1% of Rs. 37,50,000


IPCC - Income Tax

37,500

1,174,015 3,550,985

Long-term capital 18 September 2009 gain

144

32)Solution Notes :
Section 50C provides that if the value assessed by the stamp valuation authority exceeds sale consideration, then, such stamp duty value shall be deemed to be the full value of consideration. It further provides that, if the value ascertained by the valuation officer exceeds the stamp duty value, then, the stamp duty value shall be the full value of consideration. Accordingly, where section 50C is applicable and reference has been made to valuation officer, the full value of consideration will be Higher of Sale consideration or Stamp duty value; subject to maximum of value assessed by Valuation officer. Since the house has been obtained under inheritance, which is a mode specified u/s 49(1), therefore, its cost shall be the cost to the previous owner (father) i.e. FMV as on 14-1981 i.e. Rs. 2,70,000. However, for the purpose of indexation, the CII of the year of inheritance shall be used.
18 September 2009 IPCC - Income Tax 145

33)Question
Section 50C & 54EC : Mr. A who transfers land and building on 2.1.2009, furnishes the following information : Net consideration received Rs. 10 lakhs. Value adopted by stamp valuation authority, which was not contested by Mr. A Rs. )2.5 lakhs. Value ascertained by Valuation Officer on reference by the Assessing Officer Rs. 13 lakhs. This land was distributed to Mr. A on the partial partition of his HUF on 1.4.1981. Fair market value of the land as on 1.4.81 was Rs. 1,10,000. A residential building was constructed on the above land by Mr. A at a cost of Rs. 3,20,000 (construction completed on 1.12.2004) during the financial year 2004-2005. Short-term capital loss incurred on sale of shares during the financial year 2004-2005 Rs. 2,05,000. Mr. A seeks your advice as to the amount to be invested in NHAI bonds so as to be exempt from clutches of capital gain tax. Cost inflation indices for the financial years 1981-82, 2004-05 & 2008-09 are 100, 480 and 582 respectively. (Modified May 2006) (8 Marks)
18 September 2009 IPCC - Income Tax 146

33) Solution
Computation of Capital Gains of Mr. A for the Assessment Year 2009-10. (Amounts Particulars in Rs.) Rs.
Full value of consideration (Higher of Rs. 10 lakh or Rs. 12.5 lakhs; subject to maximum of Rs. 13 lakhs) Rs. 1,250,000

Less : Indexed cost of land (1,10,000 x 582 =100) Indexed cost of building (3,20,000 x 582 480) Long-term capital gains

640,200 388,000 1,028,200

221,800

Less: Brought forward short-term capital loss set off

205,000

Amount to be invested in NHAI/REC bonds to claim exemption u/s 54EC IPCC - Income Tax
18 September 2009

147

16,800

33)Solution
Note : Since acquisition of capital asset on partition of HUF is one of the modes specified in section 49(1), hence, cost thereof shall be the cost to the previous owner. Hence, FMV as on 1-4-1981 shall be the cost of acquisition and shall be eligible for indexation.

18 September 2009

IPCC - Income Tax

148

37) Question
Section 50, 51 & 54EC : Mr. Malik owns a factory building on which he had been claiming depreciation for the past few years. It is the only asset in the block. The factory building and land appurtenant thereto were sold during the year. The following details are available : (amounts in Rs.)
Building completed in September, 2004 for Land appurtenant thereto purchased in April, 2003 for Advance received from a prospective buyer for land in May, 2004, forfeited in favour of assessee, as negotiations failed WDV of the building block as on 1.4.2008 Sale value of factory building in November, 2008 Sale value of appurtenant land 1,000,000 1,200,000 50000 874,800 800,000 1,600,000

The assessee is ready to invest in long-term specified assets under section 54EC within specified time. Compute the amount of taxable capital gain for the assessment year 2009-10 and the amount to be invested under section 54EC for availing the maximum exemption. 149 IPCC - Income Tax 18 September 2009 Cost inflation indices are 2004-05 : 480; 2005-06 : 497 ; 2008-09 : 582 (Nov. 06) (10 Marks)

37) Solution
Computation of Capital Gain (in Rs.)

Sale consideration of the block Less : Cost of acquisition i.e. WDV of block

800,000 874,800

Short- term capital gains

-74,800

Sale value of land Less: Indexed cost of land (12 lakh - 50,000 advance forfeited) i.e. Rs. 11,50,000 x 582 463

1,600,000 1,445,572

Long-term capital gains

154,428 79,628

18 September 2009

Taxable long-term capital gains (after set-off of short-term capital loss) IPCC - Income Tax

150

CAPITAL GAINS EXEMPT FROM TAX [Section 10]


Sec. 10(33) 10(37) Exempted Income Capital gains on transfer of units of US 64 Any Capital Gains arising to individual or HUF from transfer of urban agricultural land by way of compulsory acquisition under any law or transfer the consideration of which is determined or approved by Central Government/RBI. Conditions/Remarks Exempt if transferred on or after 1.4.2002. Such land must have been used by individual or his parents or the HUF for agricultural purposes during two years preceding the date of transfer. Compensation or consideration for transfer (or enhanced or further enhanced compensation) is received by the assessee on or after 1.4.2004. However, the income by way of long-term capital gains of a company shall be taken into account in computing the book profits and income-tax payable under Section 115JB. (Amendment by Finance Act, 2006 w.e.f. 1.4.2007)

10(38)

Long-term capital gains arising from transfer of Equity Shares in a company or a unit of equity oriented fund, if such transaction has been charged to securities transaction tax.

18 September 2009

IPCC - Income Tax

151

COMPUTATION OF TAX ON SHORT TERM AND LONG TERM CAPITAL GAINS


Short-term Capital Gains (STCG) on transfer of an equity share of a company or a unit of an equity-oriented fund on which securities transaction tax has been charged[s.111A]: tax is computed on such capital gains at a flat rate of 15% (amendment by Finance act, 2008 w.e.f 1-4-2009). However, in case of resident individual or resident HUF, ifa) other income (i.e. total income-such STCG) is less than' basic exemption limit.' b) Then, such STCG shall be reduced by such shortfalls and c) Tax on balance of STCG shall be computed @15%. d) Accordingly, tax on such STCG = 15%*[Such STCG (basic exemption limitother income) Further, where gross total income of an assessee includes any such short-term capital gains, the deduction under chapter VIA shall be allowed from the gross total income as reduced by such gains. Other short-term capital gains: they are taxed at the normal rates applicable to the assessee.

18 September 2009

IPCC - Income Tax

152

COMPUTATION OF TAX ON SHORT TERM AND LONG TERM CAPITAL GAINS


Long - term Capital gains [ sec. 112]: tax is computed thereon at a flat rate of 20%.However, in case of resident individual or resident HUF, ifOther income (i.e. Total income -such LTCG) is less than' basic exemption limit', then, such, LTCG shall be reduced by such shortfall and Tax on balance of LPCG shall be computed@20%. Accordingly, tax on such LTCG= 20% *[such LTCG-(basic exemption limit-other income)].

Other points are:Deduction under section 80C to 80U are not available in respect of longterm capital gains. Tax payable in case of listed securities, etc. not to exceed 10%: in case of long-term capital gains arising from transfer of listed securities, units of UTI or mutual funds specified in sec 10(23D) or zero coupon bonds, the tax payable of such capital gains shall be lower of the following(i) 10% Grass Capital Gains (without indexation and without giving benefit of basic exemption limit); (ii) 20% of taxable LTCG as computed above.

18 September 2009

IPCC - Income Tax

153

Illustration 4 Capital Gains on transfer of listed securities: Mr. X bought 10,000 Equity Shares of TT Ltd. listed in stock exchange in India and abroad on 15th March, 2006 @ Rs.2,250 per share. He sold the shares at Rs.5, 000 per share on 31st December, 2008. The brokerage and securities transactions tax deducted were at 0.5% and 0.1% respectively. Examine the liability of Mr. X to income tax. Will your answer be different, if instead of selling the shares in the market, Mr. X privately transferred the shares to his son at the same price?

18 September 2009

IPCC - Income Tax

154

Solution Capital Gains on transfer of listed securities:


Tax Liability of Mr. X for the assessment year 200910: Sale transaction on which securities transaction tax has been charged: As per Section 10(38) any long-term capital gains arising out of transfer of Equity Shares in a company shall be exempt from income tax if such transaction is chargeable to securities transaction tax. Hence, in this case, LTCG will be exempt. When shares are privately transferred to his son: Since the shares are not transferred through recognized stock exchange, it will not be exempt u/s 10(38). Capital gains will be computed as under : (Amounts in Rs.)

18 September 2009

IPCC - Income Tax

155

Solution Capital Gains on transfer of listed securities:


Full value of consideration [Rs.5,000 10,000] Less : Brokerage @ 0.5% (Assuming that brokerage is payable for effecting private transfer also) Net consideration Less : Indexed cost of acquisition [2,250 10,000 519 463] 5,00,00,000 2,50,000 4,97,50,000 2,52,21,382

Long Term Capital Gains Income Tax on LTCG : [Lower of (a) or (b)] (a) 20% of (2,45,28,618 1,50,000, basic exemption limit assuming that X has no other income) (b) 10% of (4,97,50,000 2,25,00,000), benefit of basic exemption limit is not available Therefore, amount of income tax on LTCG Add : Surcharge @ 10% Income tax plus surcharge Add : Education Cess @ 2% Tax Liability of Mr. X

2,45,28,618 48,75,724 27,25,000

27,25,000 2,72,500 29,97,500 59,950 30,57,450

18 September 2009

IPCC - Income Tax

156

34) Question
Tax on LTCG u/s 112 : On 23rd December, 2008, Rajat sold 500 grams of gold, the sale consideration of which was Rs. 2,50,000. He had acquired this gold on 20th August, 1980 for Rs.40,000. Fair market value of 500 grams of gold on 1st April, 1981 was Rs. 36,000. Find out the amount of capital gain chargeable to tax for the assessment year 2009-10. Also calculate the tax liability. (CS Inter June 2006)
18 September 2009 IPCC - Income Tax 157

34)Solution
Computation of Capital Gains Tax Liability (amounts in Rs.)
Full Value of Consideration Less : Indexed cost of acquisition (Rs. 40000 x 582 =100) (Cost will he taken, as FMV is lower) 250,000 232,800
17,200 Long-term capital gain

Tax on capital gain I@ 20% Education cess @ 3%


Tax Liability

3,440 103
3,543

Note : In absence of any information in respect of other incomes of Rajat, the tax liability on long-term capital gains has been computed assuming that his other income doesn't fall short of basic exemption limit.
18 September 2009 IPCC - Income Tax 158

Provisions to curb tax avoidance by certain transactions in securities or prevention of dividend Stripping and Bonus-Stripping Transaction [sec 94].
1) Loss on sale of securities or units to be ignored in cases of dividend stripping [S.94 (7)]: If a personbuys/acquires any securities or units within a period of 3 months prior to record date, sales/transfers the same within 3 months (9 months in case of unit) after record date, and the dividend/income on such securities or unit received or receivable by him is exempt, Then, the loss, if any arising to him on account of such purchase and sale, to the extent of dividend or income from securities/unit, shall be ignored while computing his income chargeable to tax. 2) Loss arising in case of bonus stripping of units to be ignored [S. 94(8)]: in case a persona) buys/acquires any units (' original units'), within a period of 3 months prior to record date; b) He is allotted bonus units on the basis of holding of such units on such date; and c) he sells or transfers all or any of the original units referred to in (a) within a period of 9 months after such date, while continuing to hold all or any of the bonus units referred to in (b), Then(a)The loss, if any, arising to him on account of purchase and sale of original units shall be ignored in computing his total income, and (b) The loss so ignored shall be deemed to be the cost of purchase or acquisition of such bonus units referred to in (b) as are held by him on the date of such sale or transfer. Record date: record date means the date fixed by a company for entitlement of dividend, or by a mutual fund/administration/specified company for entitlement of dividend of bonus units.

18 September 2009

IPCC - Income Tax

159

35) Question
Ignoring loss in case of dividend stripping under section 94(7) : Chandu purchased units of UTI worth Rs. 10,00,000 on 1-42008. He received dividend of Rs. 1,50,000 on such units on 1-6-2008 (Record date : 31-5-2008). He sold the units on 31-12-2008 for Rs. 8,00,000. His other income under the head 'Capital Gains' is Rs. 5,00,000. Compute his total income for the assessment year 2009-10.
18 September 2009 IPCC - Income Tax 160

35)Solution Chandu has purchased units within 3 months before record


date and sold the same within 9 months after record date. The income from units is exempt u/s 10(35). So, section 94(7) will apply to him. Computation of total income of Chandu for assessment year 2009-10 Rs.
0 Income from units of UTI [Exempt under section 10(35)] Short term capital loss on purchase and sale of units [8,00,000 - 10,00,000] 200,000

Less : Exempted income from units (Loss from sale reduced by exempted income)

150,000

-50000

500,000 Other income under the head 'Capital Gains'


Total 18 income of Chandu IPCC - Income Tax September 2009 for assessment year 2009-10 450000 161

36)Question
Ignoring loss arising from bonus stripping under section 94(8) : Mohan purchased 10,000 units of mutual fund specified under section 10(23D) on 1-5-2008 for Rs. 20,00,000. He received 1,000 bonus units on the original holding of 10,000 units on 15-7-2008 (being record date). Thereafter, on 14th February 2009, he sold the original 10,000 units for Rs. 16,00,000 and retained the bonus units. Later, on 15-03-2009 he sold 600 units out of bonus units for Rs. 2,50,000. His other income is Rs. 5,00,000. Compute the total income of Mohan for assessment year 2009-10.

18 September 2009

IPCC - Income Tax

162

36) Solution
This case clearly falls within the purview of section 94(8). Accordingly, the loss of Rs. 4,00,000 [i.e. 20,00,000 - 16,00,000] incurred by Mohan on purchase and sale of units shall not be considered in computing Mohan's total income. The loss of Rs. 4,00,000 shall be deemed to be the cost of purchase of bonus units. On sale of bonus units by Mohan, the capital gains shall be computed as follows Rs.
Short-term Capital gains :

Full Value of consideration on sale of 600 units Less: Cost of acquisition of 600 units [4,00,000 x 600 1,000] Other income

250,000 240,000 10,000

500,000 Total Income 510,000


18 September 2009 IPCC - Income Tax 163

TREATMENT OF INCOME FROM DEEP DISCOUNT BONDS (DDBs) [Circular No. 2 dated 18.02.2002]
DDBs are to be valued on 31st March of each financial year. If they are held as investments, the income therefrom shall be interest income or capital gains. However, if they are held as trading assets, income therefrom shall be business income. Tax treatment of income from deep discount bonds is as follows:
1. General Treatment Interest Income/Business Income = Difference between the market valuations as on two successive valuation dates. Where bond is acquired during the year by an intermediate purchaser, Interest income/Business Income = Market Value as on valuation date Actual cost of acquisition Short-term Capital Gains/Business Income = Sale Price [Cost for which bond was acquired by the transferor + Income, if any, already offered to tax by such transferor upto the date of transfer, as per general treatment given above] Note: The capital gains arising to investors shall always be short-term capital gains. Redeemed by Original Subscriber: Interest Income/Business Income = Redemption price Value as on last valuation date immediately preceding the maturity date. Redeemed by an Intermediate Purchaser : Interest Income/ Business Income = Redemption price [Cost at which bonds were acquired by him + Income, if any, already offered to tax by the person redeeming the bond] IPCC - Income Tax 164

2. Transfer of Bonds before maturity

3. Redemption on maturity

18 September 2009

TAXABILITY OF ZERO COUPON BONDS [ZCBs]


(i) According to Sec. 2(48), Zero Coupon Bonds means a bond issued by any infrastructure capital company or infrastructure capital fund or public sector company on or after the 1st day of June, 2005; in respect of which, no payment and benefit is received or receivable before maturity or redemption from infrastructure capital company or infrastructure capital fund or public sector company; and Which the Central Government may, by notification in the Official Gazette, specify in this behalf. (ii) Any maturity or redemptioin of ZCBs shall be treated as transfer as per Section 2(47) and accordingly subject to tax under the head Capital Gains. However, in case such bonds are held as stock-in-trade of the business, it shall be chargeable to tax under the head Profits and Gains of Business or Profession. (iii) Any long term capital gain arising from the transfer of ZCBs shall be entitled to a concessional tax rate of 10% without the benefit of indexation or at the rate of 20% after availing the benefit of indexation. (iv) In case ZCBs are held for less than 12 months, it shall be considered as short term capital asset u/s 2(42A). However, concessional rate of 10% tax provided for short-term capital gains on transfer of listed shares u/s 111A is not applicable to ZCBs. Therefore, short term capital gains on transfer of ZCBs shall be subject to tax as per normal rates of tax.
18 September 2009 IPCC - Income Tax 165

SPECIAL PROVISIONS FOR NON-RESIDENTS


In the case of an assessee who is a non-resident, capital gains arising from transfer of capital assets being the shares or debentures of an Indian company shall be computed by converting cost of acquisition, expenses incurred for the transfer and sale consideration into the same foreign currency as was utilized for the purchase of shares or debentures as indicated below. The capital gains so computed in such foreign currency shall be reconverted into Indian currency for the purpose of further computation First proviso to Section 48 and Rule 115A.
Items Converted/ Reconverted 1. 2. 3. 4. Cost of acquisition Expenses incurred for transfer Sale consideration Capital Gains [Reconversion] Rate of Conversion/Reconversion

The average of telegraphic transfer selling rate and buying rate as on the date of acquisition The average of telegraphic transfer selling rate and buying rate as on the date of transfer The average of telegraphic transfer selling rate and buying rate as on the date of transfer The buying rate for telegraphic transfer as on the date of transfer

18 September 2009

IPCC - Income Tax

166

SPECIAL PROVISIONS FOR NON-RESIDENTS The conversion and reconversion shall be made on the basis of the rate of exchange adopted by the State Bank of India. The aforesaid manner of computation of capital gains shall be applicable in respect of capital gains arising from every reinvestment thereafter in the shares or debentures of an Indian company on the sale of such assets. In these cases, indexation will not be available in the computation of capital gains.

18 September 2009

IPCC - Income Tax

167

Illustration:
Mr. Fedrick, a non-resident Indian, acquired in January, 2002, shares in Indian companies for a consideration of Rs.20.50 lakhs by remitting equivalent US Dollars. In October, 2006, he sold the entire shares for a sum of Rs.33, 00,000 after incurring Rs.66, 000 towards expenses for transfer. You are informed the details of telegraphic transfer rates of State Bank of India herebelow: ParticularsBuying rateSelling rateOn the date of acquisitionOn the date of transfer40.5043.5041.5044.50 Compute the taxable capital gains on the basis of the above information.

18 September 2009

IPCC - Income Tax

168

Solution :
Computation of Long Term Capital Gains for the assessment year 2007-08
Particulars Indian Rupees Rate of Conversio n 44.00 44.00 US $

Sale consideration Less : Expenses for transfer

33,00,000 66,000

75,000 1,500

Net consideration Less : Cost of acquisition

20,50,000

41.00

73,500 50,000

Capital Gain Assessable

10,22,250

43.50

23,500

18 September 2009

IPCC - Income Tax

169

Indian, remits US $80,00,000 to India on 16th September, 1989. The amount is partly utilised on 3rd October, 1989 for purchasing 10,000 shares in Amrit Ltd., an Indian company, at the rate of Rs. 12 per share. These shares are sold for Rs. 38.40 per share on 28th February, 2009. Rakshit deposited Rs. 1,92,000 with an Indian public company on 1st August, 2009 (due date of filing of return of income 31st July, 2009). You are required to compute the capital gains chargeable to tax for the assessment year 2009-10 on the assumption that telegraphic transfer buying and selling rate of US dollars adopted by the State Bank of India is as follows :

31) Question Capital gains under 1st proviso to section 48 & Rule 115A : Rakshit, a non-resident

Telegraphic.Transfer Rate Buy a US $ (Rs.) 18 19 44 Sell a US$ (Rs.) 20 21 46


170

16th September, 1989 3rd October, 1989 28th February, 2009


18 September 2009

(CS Dec. IPCC - Income Tax

2007) (7 Marks)

31)Solution
Computation of capital gains chargeable to tax for the Assessment Year 2009-10
US $ Full value of consideration (38.40 x 10,000 + 45) (Avg. exchange rate on date of transfer i.e. 28-2-09) Less: Cost of acquisition (12 x 10,000 + 20) (TT buying rate on date of purchase i.e. 3-10-1989)

8533.33 6000.00

2533.33 Capital Gains Capital gains (in Indian Rs:) = 2,533.33 x Rs. 44 (TT buying rate on date of transfer i.e. 28-2-09) = Rs. 1,11,467.
18 September 2009 IPCC - Income Tax 171

38) Question
Capital gains and tax on various kinds of shares : Abhay purchased 1,000 listed equity shares of Rs.10 each at Rs.100 per share from a broker on 4th May, 1996. He paid Rs. 3,000 as brokerage. On 15th March, 2008, he was given bonus shares by the company on the basis of one share for every two shares held. On the same date, he was also given a right to acquire 1,000 rights shares @ Rs. 90 per share. He acquired 50% of the rights shares offered and sold the balance 50% of the rights shares for a sum of Rs. 67,500 on 7th April 2008. The rights shares were allotted to him on 30th April, 2008. All the shares held by him were sold on 24th September 2008 @ Rs. 280 per share through a recognised stock exchange. Compute capital gain and tax assuming his income from other sources is Rs. 40,000. (CS Inter June 2005)

18 September 2009

IPCC - Income Tax

172

38) Solution : (1) The original equity shares are a long-term capital asset. Since they
have been sold through a recognised stock exchange, hence, the long-term capital gains arising therefrom will be exempt u/s 10(38). (2) Capital gains on shares other than original shares (amounts in Rs.)
, Rights (500) Sale consideration (@ Rs. 280 per share) Less : Cost of acquisition Short-term capital gains Total short-term capital gains falling u/s 111A (from sale of right Other short-term capital gains Income from other sources Total Income Tax [15% of (2,35,000 - (1,50,000, being basic exemption limit - 40,000 - 67,500))] (Note) Add : Education Cess and SHEC @ 3% Tax liability 28875 866 29,741 67500 40,000 342500 67500 NIL 67500 Right shares (500) Bonus shares. (500) 140000 45000 95000 and bonus shares) 140000 NIL 140000 235000

Note : The other income (i.e. the income other than STCG falling u/s 111A) of the assesseee is Rs. 67500 + 40000 = Rs. 1,07,500. It is less than bais exemption limit of Rs. 1,50,000 by Rs. 42500. Therefore, the amount of taxable STCG falling u/s 111A = 2,35,000 - 42500 = Rs. 192500. Tax thereon @ 15% = Rs. 28875. 173 IPCC Income Tax 18 September 2009

39) Question
Capital gains & Total Income : Mr. Y submits the following information pertaining to the year ended 31st March, 2009 : On 30.11.2008, when he attained the age of 60, his friends in India gave a flat at Surat as a gift, each contributing a sum of Rs.20,000 in cash. The cost of flat purchased using various gifts was Rs. 3.4 lacs. His close friend abroad sent him a cash gift of Rs. 75,000 through his relative, for the above occasion. Mr. Y sold the above flat on 30.1.2009 for Rs.3.6 lacs. The Registrar's valuation for stamp duty purposes was Rs.3.7 lacs. Neither Mr. Y nor the buyer, questioned the value fixed by the Registrar. He had purchased some equity shares in X Pvt. Ltd. on 5.2.2008 for Rs. 3.5 lacs. These shares were sold on 15.3.2009 for Rs.2.8 lacs. You are requested to calculate the total income of Mr. Y for the assessment year 2009-10 [Cost Inflation Index for F.Y. 2007-08 : 551 and F.Y. 2008-09 : 582]. (May 2005) (6 Marks)

18 September 2009

IPCC - Income Tax

174

39) Solution
Computation of total income of Mr.Y for A.Y.2009-10 (amounts in Rs.)

Flat* Full value of consideration (*Stamp duty value, as it is higher - Section 50C) 370,000

Equity shares** 280,000

Less : Cost (*Gift is section 49(1) mode, cost = cost to previous owner) (**Indexed cost of shares = 350000 x 582 551)

340000

369,691 Short-term capital gains 30,000 Long-term capital loss to be carried forward Income under the head capital gains (Short-term capital gains) Income from other sources (gift from non-relative exceeding Rs. 50,000) 18 September 2009 IPCC - Income Tax TOTAL INCOME 30,000 75,000 105,000 175 89,691

39) Solution
Notes : Gift of flat by friends is not taxable. Further, on sale of the same, the cost thereof to the assessee shall be the cost to the previous owner (friends, here). Cash gift exceeding Rs. 50,000 from an unrelated person is taxable as Income from other sources. Equity shares held for more than 12 months from the date of purchase are long-term capital asset. Hence, benefit of indexation shall be available. However, since the equity shares are of a private company, therefore, the gains arising therefrom shall not be exempt u/s 10(38).
18 September 2009 IPCC - Income Tax 176

40)Question
Gains on various assets : Mrs. Padmini owned two motor cars which were mainly used for business purposes. The written down value on 1-4-2008 of the block of assets comprising of only these two cars, both of which were purchased in May 2001, was Rs. 1,81,000. These two cars were sold in June, 2008 for Rs. 1,50,000. In February 2009, she sold 1,000 shares in X Ltd., an Indian Company, for Rs. 3,50,000 through a recognised stock exchange. She had purchased the same during March, 2004 for Rs. 2,44,000. A house plot purchased by her in March, 2002 for Rs. 2,73,000 was sold by her for Rs. 5,00,000 on 18-1-2009. Compute the amount of net capital gains chargeable to tax in respect of the above transactions for the assessment year 2009-10. (Nov. 1993)

18 September 2009

IPCC - Income Tax

177

40) Solution Long-term capital gains arising from sale of equity shares
through recognised stock exchange are exempt under section 10(38). The other capital gains chargeable to tax are (amounts in Rs.) -

. Motor cars House plot

Sale consideration Less : Cost (WDV of cars; Indexed cost of house = 2,73,000 x 582 Short-term capital loss Long-term capital gains 426)

150,000 181,000 31,000

500,000 372,972

127,028

Income under the head capital gains

18 September 2009

IPCC - Income Tax

178

96,028

41) Question
Alteration of share capital : In April, 2001, S subscribed to the first issue of equity capital of a public limited company (face value of each share was Rs. 100) to the extent of Rs. 25,000. In 2005, the company converted the face value of its shares from Rs. 100 to Rs. 10 each. Half of the holdings of the shares held by S was sold by him in October, 2008 for Rs, 50,000. S had to pay a brokerage of 2% on sale. What is the nature of gains realised and compute the same. (May 1998)
18 September 2009 IPCC - Income Tax 179

41) Solution
Solution : Computation of Capital Gain for the Assessment Year 2009-10 (amounts in Rs.)

Consideration price of 50% shares i.e. 1250 shares (See Note) Less: Expenses of transfer i.e. brokerage @ 2%
Indexed cost of acquisition (12,500 x 582 426)

50,000 1,000 17,077 31,923

Long-term capital gains

Note : No. of original shares = 250 shares of Rs. 100 each. On sub-division of share capital from face value of Rs. 100 to Rs. 10, the total number of shares = 2500 shares of Rs. 10 each. It is assumed that the sale is not through a recognised stock 180 - Income Tax would have been exempt. 18 Septemberotherwise 2009 exchange, the long-term IPCC capital gains

42) Question
Conversion or redemption of debentures : Amin is the holder of 1,000 debentures of Amin Ltd. having a face value of Rs. 1,000 each. The company has offered an option to the debenture-holders either to redeem the debentures at Rs. 1,200 each or to convert the debentures into equity shares of equivalent value. The market value of the shares on the date of exercising the option is Rs. 1,200 per share (face value Rs. 1,000). What will be the tax consequences of the two options in the hands of the debenture-holder Amin? (May 1997)

18 September 2009

IPCC - Income Tax

181

42)Solution
(1) Redemption : The first option of redemption of debentures will result in the levy of capital gains tax. Capital gains = (1200 - 1000) x 1000 debentures = Rs. 2 lakh. Indexation benefit is not allowed. (2) Conversion : There will be no capital gain on conversion of debentures into equity shares, as the 'conversion of debenture into shares is not 'transfer' as per section 47(x).
18 September 2009 IPCC - Income Tax 182

43)Question
Taxability of Sweat equity shares : Under a scheme framed on 1-1-2008 for allotment of Sweat Equity Shares, XYZ Info Ltd. (an unlisted company) allotted sweat equity shares to its employees on 1-5-2008 at Rs. 100 per share. The fair market value of the sweat equity shares on 28-3-2008 (i.e. on the date of vesting of option in employees) was Rs. 300 per share, as per rules. Mr. P, an employee who fell under such scheme, was allotted 1,000 shares. Compute the tax implications of these transactions, given that Mr. P left XYZ Ltd. and sold the shares allotted to him on 31-3-2009 for Rs. 450 per share.
18 September 2009 IPCC - Income Tax 183

43)Solution
Allotment of sweaty equity shares attracts liability to Fringe Benefit Tax (FBT) in the hands of the company to the extent the value of fringe benefit. Value of fringe benefit = FMV on the date of vesting of option - Amount recovered from Mr. P = (300 - 100) x 1,000 = Rs. 2,00,000. FBT payable by company = 2,00,000 x 33.99% (30% + 10% Surcharge + 3% EC & SHEC) = Rs. 67,980. Short-term Capital gains in hands of Mr. P = (450 - 300) x 1,000 = Rs. 1,50,000. Note : The period of holding shall be computed from the date of allotment of sweat equity shares, which is not more than 12 months i.e. shares are short-term capital asset. Its cost shall be the fair market value taken for assessing fringe benefits. The implications under company law and SEBI laws have been ignored.

18 September 2009

IPCC - Income Tax

184

44)Question
Capital gains on sale of convertible debentures with securities received on conversion : Mr. Sohan had acquired 100 Partly Convertible Debentures of a listed company on 15` April 2006 for Rs. 1,000 each. On 15t April 2008 Mr. Sohan received 1000 equity shares on conversion of 40% portion of each such debenture into 10 equity shares. On 30th September 2008, he sold 500 equity shares @ Rs. 150 each and 50 debentures @ Rs. 1800 per debenture. The brokerage for sale of shares and debentures was 1% and Securities Transaction Tax @ 0.1% was charged on the sale of equity shares. Compute capital gains.
18 September 2009 IPCC - Income Tax 185

44)Solution
Computation of tax liability of Mrs. K for the assessment year 2009-10 (amounts in Rs.)
Debentures Equity Shares

Sale Price[(50 x 1800); (500 x 150)]


Less: Expenses on transfer (1% of sale price, STT not deductible) Less: Cost of Acquisition [(50 x 600);(500 x 40)] [Note 1 to 3] Long-term Capital Gains

90,000 900 30,000

75,000 750 20,000

59,100

Short-term Capital Gains


18 September 2009 IPCC - Income Tax

54,250
186

44) Solution
Note: Investment in bonds of NABARD doesn't qualify for exemption. Further, investment in bonds of NHAI will be not be eligible for exemption as investment is not made within 6 months time limit.

18 September 2009

IPCC - Income Tax

187

45) Question
Capital gains tax on capital indexed bonds : Mrs., K had purchased 40 Capital indexed bonds listed in recognised stock exchange of Rs. 10,000 each on 1-4-2007. She sold all the bonds @ Rs. 20,250 on 28th September 2008. She invested Rs. 80,000 in bonds of NABARD on 1-11-2008 and Rs. 70,000 in the bonds National Highway Authority of India on 31' March 2009. Compute the tax liability of Mrs. K assuming that she has no other income chargeable to tax.
18 September 2009 IPCC - Income Tax 188

45)Solution
Sale Price [(20,250 x 40)] Less: Indexed Cost [(40 x 10,000 x 582 551)] Long-term capital gains Total Income (rounded off) Tax on LTCG Rs. 387500 (rounded off to nearest Rs. 10), being lower of the following (a) 20% of (LTCG - 180000) (b) 10% of LTCG (without indexation & without basic exemption) i.e. 10% of [(20250 - 10000) x 40) Add: Education Cess and Secondary and higher education Cess @ 3% Tax Liability 810,000 422,505 387,495

387,500

41,500 41,000

41,000 1,230 42,230

Note: Investment in bonds of NABARD doesn't qualify for exemption. Further, investment in bonds of NHAI will be not be eligible for exemption as investment is not made within 6 months time limit.
18 September 2009 IPCC - Income Tax 189

46) Question Comprehensive Illustration

on computation of capital gains along with planning for exemption under section 54F : On lst September 2008, Mr. X sold the following assets (all assets are long-term capital assets and all amounts are in Rs.): -

Asset

Unlisted debentures

Gold Plot of Land

Diamonds

Sale Price

500,000
Cost/Indexed Cost
18 September 2009

2,000,000

1,000,000 1,200,000

275,000

1,400,000
IPCC - Income Tax

650,000

700,000
190

46) Question
Other information : The brokerage and other expenses on transfer of each asset may be taken at 1% of sale price thereof. On 8th February 2009, X acquired 70 bonds of National Highway Authority of India of Rs. 3,50,000; and purchased a residential house property of Rs. 26,00,000. Compute the amount of capital gains chargeable to tax.
18 September 2009 IPCC - Income Tax 191

46) Solution Computation of amount of capital gains chargeable


to tax for the assessment year 2009-10:
Plot of land Unlisted debentures Full Value of Consideration [Note 2] Less: Brokerage etc. 1% of sale price Net consideration Less: Cost / Indexed Cost Long-term capital gains Total long-term capital gains Less : Exemption u/s 54EC Less : Exemption u/s 54F (See Note) 18 September 2009 IPCC - Income Tax Taxable Long-term capital gains 500,000 5,000 495,000 275,000 220,000 2,000,000 20,000 1,980,000 1,400,000 580,000 1,000,000 10,000 990,000 650,000 340,000 1,200,000 12,000 1,188,000 700,000 488,000 1,628,000 350000 1,022,929 255,071 192 Gold Diamonds

46) Solution
Note : Since the exemption u/ s 54F is based on formula (Cost of new asset x LTCG Net consideration of the asset sold), therefore, the exemption availed will be highest in the case of those assets where percentage of LTCG to Net Consideration is highest. The cost of new asset will be utilised first of all against the asset whose percentage so computed will be highest and so on. For this purpose, the cost of new house will be apportioned between various assets on the basis of the priority. Accordingly, the relevant computations are as under Unlisted debentures Plot of land Gold Diamonds

220,000 (A) Long-term capital gains (B) Net Consideration [See Note Below] Capital gains as % of net consideration Priority (C) Total cost of new house Rs. 26 lakhs allocated as per priority towards investment of net consideration of the asset 495,000 44.44%

580,000 1,980,000 29.29%

340,000 990,000 34.34%

488,000 1,188,000 41.08%

495,000 220,000 -

917,000 314,929

1,188,000 488,000 193 1,022,929

Exemption u/s 54F (residential house) (A x C B) 18 September 2009 Total exemption

IPCC - Income Tax

47)Question
Salary & Capital gains with exemption u/s 54F : Rajesh, a director at Young Pvt Ltd., Mumbai, carrying on transport business has submitted the following particulars of his income for the assessment year 2009-10. Compute his taxable income and tax payable: (1) Director's remuneration and perquisites : Remuneration @ Rs. 11,000 p.m. : Rs. 1,32,000. Free use of car of engine capacity of 1,500 cc provided by the company. The company has provided rent-free accommodation (unfurnished) to Rajesh at Mumbai. It is ascertained that for this purpose the company has obtained a flat on lease from Prem Ltd. for which the company pays a rent of Rs. 40,000 p.a., which is also the fair rental value. Medical expenses met by the company on an ordinary treatment of Rajesh and the members of his family Rs. 4,500. (2) Rajesh was occupying a bungalow on rent at New Delhi since November 1986. He agreed to transfer his tenancy right in the said bungalow in favour of Bala Ltd. for a sum of Rs. 2,00,000 payable as follows: Rs. 50,000 in cash. A flat valued at Rs. 1,50,000 (at cost) in the new building to be put up by Bala Ltd to be allotted free of cost. The cash payment was made on 5th April, 2008 and the flat was allotted to him on 5th October, 2008 which was kept for his own residence from that date. (CS Inter Dec. 2003)
18 September 2009 IPCC - Income Tax 194

47) Solution
Computation of taxable income for the assessment year 2009-10 Income from salaries : Rs.
Remuneration (11000 x 12) Use of car (liable to FBT, hence, exempt for the employee) Value of residential accommodation, being the lower of (a) 15% of salary (b) Lease rent Medical expenses (Exempt upto Rs. 15,000) Income under the head "Capital Gains" : (See Notes) 19,800 40,000 19,800 Exempt 151,800 Rs. Rs.

132,000 Exempt

18 September 2009

IPCC - Income Tax

195

47) Solution
Long-term capital gains from transfer of tenancy rights Less : Exemption u/s 54F (1,50,000 x 2,00,000 x 2,00,000) Total income Tax on total Income [20% of LTCG + Normal tax on other income i.e. 20% x 50000 + (151800 - 150000) x 10%] Add : EC and SHEC @ 3% Tax payable
18 September 2009 IPCC - Income Tax

200,000 150,000 50,000 201,800

10,180 305

10,485
196

47)Solution
Notes : Tenancy right is a capital asset. Since it has been held from November, 1986, it is a long-term capital asset. Its cost of acquisition is NIL. Therefore, the net consideration from transfer and the amount of long-term capital gains, both, are Rs. 2 lakh. Rajesh has been allotted a flat on the transfer of tenancy rights. This amounts to transfer of tenancy right and investment of the consideration received therefrom for the purchase of flat. This investment shall qualify for exemption u/s 54F, the amount of exemption being : (Cost of flat x LTCG Net consideration from the transfer). Thus, the exemption will be Rs. 1.5 lakh.
18 September 2009 IPCC - Income Tax 197

You might also like